aamc mcat test 8 solnsdocshare01.docshare.tips/files/23478/234782350.pdf · 2016. 12. 13. · mcat...

41
SOLUTIONS MCAT * Practice Test 8 MCAT Medical College Admission Test www.PrincetonReview.com AAMC

Upload: others

Post on 14-Jun-2021

49 views

Category:

Documents


9 download

TRANSCRIPT

Page 1: AAMC MCAT Test 8 Solnsdocshare01.docshare.tips/files/23478/234782350.pdf · 2016. 12. 13. · MCAT Practice Test 8 Solutions 4 this fact from nuclear physics, you can still choose

SOLUTIONS

MCAT *

Practice Test 8

MCATMedical CollegeAdmission Testwww.PrincetonReview.com

AAMC

Page 2: AAMC MCAT Test 8 Solnsdocshare01.docshare.tips/files/23478/234782350.pdf · 2016. 12. 13. · MCAT Practice Test 8 Solutions 4 this fact from nuclear physics, you can still choose

MCAT Practice Test 8 Solutions2

MCAT Practice Test 8 SOLUTIONS

Edited, produced, typeset, and illustrated by Steve Leduc Director, MCAT Program Development The Princeton Review

CONTENTS PAGE

Physical Sciences solutions .....................................................3 Steve Leduc (Physics) Steve Leduc, Bethany Blackwell (General Chemistry)

Verbal Reasoning solutions ...................................................12 Jennifer Wooddell

Biological Sciences solutions ................................................33 Judene Wright (Biology) Bethany Blackwell (Organic Chemistry)

Copyright © 2005 by Princeton Review, Inc.All rights reserved.

*MCAT is a registered trademark of the Association of American Medical Colleges (AAMC).The Princeton Review is not affiliated with Princeton University or AAMC.

This document is for the exclusive use of Princeton Review course students,and is not legal for resale.

www.PrincetonReview.com

050525

Page 3: AAMC MCAT Test 8 Solnsdocshare01.docshare.tips/files/23478/234782350.pdf · 2016. 12. 13. · MCAT Practice Test 8 Solutions 4 this fact from nuclear physics, you can still choose

Physical Sciences Solutions 3

PHYSICAL SCIENCES SOLUTIONS

Passage I (Questions 1–5)

1. A. Since sulfur and fluorine are both nonmetals, we would expect a compound of S and F to be covalent (since, as mentioned in the passage, “the vast majority of covalent compounds are comprised exclusively of nonmetallic elements”). This eliminates choices B and D. Furthermore, the passage also tells us that aqueous solutions of covalent compounds do not conduct electricity. Therefore, the answer is A.

2. A. Metals are found on the left side of the periodic table, nonmetals on the right side.

3. D. The passage states that “the vast majority of covalent compounds are comprised exclusively of nonmetallic elements.” Since both hydrogen and bromine are nonmetals, HBr is the only compound in the answer choices that can be the covalent one, so the answer must be D.

4. B. The binary compound with the most ionic character will be the one whose constituent elements have the greatest electro-negativity difference. Since Cs is directly below Rb in the periodic table, we expect its electronegativity to be no greater than that of Rb. So, the electronegativity difference for the elements in CsCl (choice B) will be no less than 3.0 – 0.8 = 2.2. No pair of elements appearing in a compound in any of the other three choices has an electronegativity difference as great as this. (For choice A, KBr, the electronegativity difference is 2.8 – 0.8 = 2; for choice C, NaI, the difference is 2.5 – 0.9 = 1.6; and for choice D, RbBr, the difference is 2.8 – 0.8 = 2.)

5. C. The passage states that “aqueous solutions of ionic compounds conduct electricity.” So, the fact that a 1 M solution of HCl conducts electricity would be consistent with the claim that HCl is ionic.

Passage II (Questions 6–12)

6. A. The last sentence of the third paragraph of the passage states that “B3 is the vector sum of the magnetic fields of electrons and other nuclei in the vicinity of the H atom” [emphasis added]. Therefore, B3 is determined by the environment of the nucleus, not by its (intrinsic) mass, radius, or charge.

7. C. The first sentence of the last paragraph of the passage tells us that “if μ is antiparallel to B1, the H nucleus eventually will relax . . . and emit energy . . .” [emphasis added], so choice C (not D) is correct. Choice A is wrong because weʼre told in the third paragraph that each H atom has a nonzero μ (and only nuclei with a net spin of zero have μ = 0). Choice B is eliminated because the magnitude of the externally produced (and controlled) static magnetic field B1 is not affected by μ.

8. B. The third paragraph of the passage begins by stating that “MRI uses magnetic fields to produce high-contrast images of human tissue.” We are told in the second paragraph that dyes often must be injected in order to enhance contrasts in images. Clearly, then, dyes may be used (even used often), but they donʼt have to be used, so choice B is better than C or D. As for choice A, the passage contains no discussion of toxicity (and this choice begs the question, “Less toxic than what?”).

9. C. The second paragraph of the passage discusses contrast. We are told that contrast results from differences in attenuation, and that attenuation coefficients are roughly proportional to the atomic numbers of the elements contained in the tissue. If the contrast is too low, this means that differences in attenuation are too low, which immediately implies that differences in the atomic numbers of the elements comprising the target tissue are too low. This is directly stated in choice C.

10. C. The formula for ωd is given in the fourth paragraph of the passage: ωd = 4πμB1/h. If B1 is nonzero, then the nuclear magnetic dipole moment, μ, must be nonzero in order for ωd to be nonzero. So, the question becomes, Which one of the four nuclei given as choices here has a nonzero μ? The magnetic moments of paired nucleons cancel each other out, so any nucleus with an even number of protons (even Z) and an even number of neutrons (even N) will have μ = 0. But even if you didnʼt know

Page 4: AAMC MCAT Test 8 Solnsdocshare01.docshare.tips/files/23478/234782350.pdf · 2016. 12. 13. · MCAT Practice Test 8 Solutions 4 this fact from nuclear physics, you can still choose

MCAT Practice Test 8 Solutions4

this fact from nuclear physics, you can still choose the correct answer. All of the choices have an even Z (and an even A, which implies an even N), except choice C, which has an odd Z. We are told in the passage that each H atom, which we know has an odd number of protons (namely, 1) has a nonzero μ. We conclude, then, that nucleus C, the only choice with an odd number of protons has, like an H atom, a nonzero μ.

11. A. The formula for ωd is given in the fourth paragraph of the passage: ωd = 4πμB1/h. For a given nucleus, μ is fixed, so ωd varies only with B1 (all other quantities in the formula are constants). Thus, B1 is the only quantity that a diagnostician could alter to adjust ωd of H nuclei.

12. C. As stated in the fourth paragraph of the passage, “If the rotational frequency of B2 equals ωd (a condition called resonance), the nucleus can become antiparallel to (make a 180° angle with) B1.”

Passage III (Questions 13–17)

13. C. To see the effect of temperature on the reaction rate, we would want all other variables to remain unchanged. Tube 5 begins with 20 mL of Solution A, 20 mL of Solution B, and nothing else. Of the other trials listed in Table 1 of the passage, only Tubes 1 and 4 have these same starting materials (note that an additional compound was added to Tube 6, which therefore changed the initial composition in this trial). Since Tube 1 is not listed among the choices, we select Tube 4 (choice C).

14. D. The reactions shown in the passage include I–, S2O32–, and S2O8

2– as reactants, so we would expect that adding any of the compounds listed in choices A, B, or C would affect the results. However, the passage tells us that starch is used only as an indicator, so it is not an active participant in the reactions under study. Therefore, adding starch to Solution B would most likely not affect the experimental results.

15. D. Table 1 shows that the trials using Tube 1 and Tube 6 had the same starting conditions: the same volume of Solution A, the same volume of Solution B, the same volume of water, and the same temperature. However, the time measured for Tube 6 is a full 10 seconds less than the time measured for Tube 1. Since the only difference between Tubes 1 and 6 was the addition of CuSO4 to Tube 6, and the result was an marked increase in reaction rate, we conclude that CuSO4 was a catalyst.

16. B. First, we can eliminate choices C and D, because according to Table 1, the initial volumes of the solutions for the trials using Tubes 1 and 4 were the same. Choice A is backwards. The passage tells us that the solution turns dark blue when one of the reactants of Reaction 2, namely S2O3

2–, gets used up. So, if the rate of Reaction 2 were slower in Tube 1 (as claimed in choice A), then the solution in Tube 1 would have taken longer to turn dark blue. The answer is B. Tube 1 was run at a higher temperature than Tube 4, so the average kinetic energy of the reactants was greater in Tube 1; this greater kinetic energy led to the faster reaction rate.

17. B. Reaction 2 proceeds until one of its reactants (S2O32–) is used up and the solution turns dark blue. According to Table

1, this reaction proceeds in Tube 6 for 19 seconds. At this point, then, no more of the product S4O62– will be formed. So, the

amount of S4O62– should rise until t = 19 seconds and remain constant thereafter. This is shown by the graph in choice B.

Passage IV (Questions 18–22)

18. B. The last two sentences of the passage tell us that the ratio of d to t2 increased as the angle of inclination increased. Therefore, this ratio should have attained its maximum value when the angle of inclination reached its maximum value—that is, when the inclined plane was vertical.

19. A. Aristotleʼs theory of natural motion deals with objects moving through the air at a constant velocity, so choice B is eliminated. A mass on a spring is acted upon by a force outside the object (namely, the force from the spring) and thus, according to Aristotle, undergoes violent motion. This eliminates choice C. The statement in choice D is also dealt with by Aristotleʼs theory; in fact, the first sentence of the passage states that “Aristotle believed that the time it takes for an object to

Page 5: AAMC MCAT Test 8 Solnsdocshare01.docshare.tips/files/23478/234782350.pdf · 2016. 12. 13. · MCAT Practice Test 8 Solutions 4 this fact from nuclear physics, you can still choose

Physical Sciences Solutions 5

fall a given distance is inversely proportional to its weight,” which would imply that a heavy body should fall faster than a lighter one. The answer must be choice A. Aristotleʼs theory of natural motion says that “an object in motion is propelled at a constant velocity by the air....” This theory would therefore not be able to explain an object moving with constant velocity in the absence of air.

20. C. First, we eliminate choices B and D because theyʼre not true. The experiments of Galileo that are described in the passage were used to measure d (distance traveled) and t (the time of travel) and then determine the ratio d/t2. It seems easy enough to measure a distance of 10 m with a reasonable degree of accuracy. However, it would have been far more difficult for Galileo to measure very short time intervals (less than, say, a couple of seconds) with a comparable degree of accuracy. (A sphere dropped from a height of 10 m would reach the ground in less than a second and a half.) Furthermore, any error incurred in measuring t would be amplified because t is squared in the ratio Galileo was calculating. Therefore, the main advantage in using an inclined plane (whose angle could be made as small as desired, thereby making the sphereʼs travel time as long as desired) is that the spheres would take longer to reach the bottom. Note that while choice A is generally true, Galileo was not measuring the final velocity of the spheres, only the distance traveled and their travel time, so choice C is a better response than A.

21. B. The first sentence of the passage states that “Aristotle believed that the time it takes for an object to fall a given distance is inversely proportional to its weight.” Therefore, since all the objects are dropped from the same height, the object with the greatest weight (which is one with the greatest mass), Sphere B, would fall to the ground the fastest, so it would have been the one that achieved the greatest terminal velocity.

22. D. The first sentence of the passage states that “Aristotle believed that the time it takes for an object to fall a given distance is inversely proportional to its weight.” Since weight is directly proportional to mass, it follows that Aristotleʼs theory would predict that the time it takes for an object to fall a given distance is inversely proportional to its mass. Letʼs compare the masses of Objects A and B described in the question. Since mass = density × volume, and Object A has twice the density but ½ the volume of Object B, Object A will have the same mass as Object B. (Note that this eliminates choices B and C.) Because their masses are the same, these spheres should therefore take the same amount of time (choice D) to fall a given distance, according to Aristotle. Note that choice A is a trap: Aristotleʼs theory about the time required for an object to fall a given distance depends on the objectʼs weight, not its density.

Free-Standing Questions (23 through 26)

23. D. The solubility equilibrium of M(OH)2 is given by M(OH)2 M2+ + 2 OH–, so if [M2+] = S, then [OH–] = 2S. The solubility product is given by Ksp = [M2+][OH–]2, which thus becomes S(2S)2 = S(4S2) = 4S3.

24. C. Treating the gas as an ideal gas, weʼll invoke the Ideal-Gas law, PV = nRT. Since the quantity of gas remains unchanged, n is a constant. Therefore, PV ∝ T, so if T increases, the product PV increases. Which of the choices correctly describes the behavior of V if the product PV increases? Only choice C: If PV increases and P is constant, then V must increase. (This also shows why choice D is wrong.) Choice A is eliminated because if P increases by a greater factor than T increases, then V would decrease. And choice B is wrong because V cannot be inversely proportional to T if PV ∝ T.

25. B. When an object floats, its weight is balanced by the buoyant force. The weight of an object with total volume V is given by mg, which is ρobjVg. The buoyant force is given by ρfluidVsubg, where Vsub is the volume that is submerged. Therefore, for an ice cube floating in a soft drink, we have ρiceVg = ρdrinkVsubg, which, after canceling the gʼs on both sides, becomes ρiceV = ρdrinkVsub, so Vsub/V = ρice/ρdrink. Since the gravitational acceleration g cancels out of the equation, it wonʼt matter whether the astronaut is on the surface of the moon or on the surface of the earth. The fraction of the ice cube thatʼs submerged, Vsub/V, is ρice/ρdrink, independent of the local value of g.

26. C. The ability of a long glass fiber to transmit light over a long distance with a minimum loss of energy is due to the phenomenon of total internal reflection.

Page 6: AAMC MCAT Test 8 Solnsdocshare01.docshare.tips/files/23478/234782350.pdf · 2016. 12. 13. · MCAT Practice Test 8 Solutions 4 this fact from nuclear physics, you can still choose

MCAT Practice Test 8 Solutions6

Passage V (Questions 27–31)

27. A. The passage tells us about “the high degree of solubility of CaCO3 in acidic solutions.” Therefore, if Reaction 1 is effective at decreasing the hardness of an acidic water sample, we would expect that it increases the pH, to make CaCO3 less soluble (and, in fact, the passage also states that as a result of Reaction 1, calcium carbonate precipitates). We therefore eliminate choices B and D. Since water hardness is directly related to the amount of dissolved CaCO3, decreasing the hardness of water means decreasing the solubility of CaCO3. This is directly stated in choice A.

28. A. Atmospheric CO2 reacts with water to produce carbonic acid, H2O + CO2 → H2CO3. This would increase the acidity of the water (that is, lower the pH), thereby allowing a greater amount of CaCO3 to dissolve (since the passage mentions “the high degree of solubility of CaCO3 in acidic solutions”). Therefore, choice A is the answer.

29. C. At STP, 1 mole of an ideal gas occupies 22.4 L, so if the evolved CO2(g) occupies 11.2 L, we know that this represents 0.5 mole of CO2. From Reaction 1, which is balanced, we can see that for each mole of CO2(g) produced, 1 mole of CaCO3(s) is produced. Therefore, 0.5 mol of CaCO3 is formed. Since the molar mass of CaCO3 is 40 + 12 + 3(16) = 100 g/mol, 0.5 mol of CaCO3 has a mass of 50 g.

30. B. The solubility equilibrium here is CaCO3(s) Ca2+(aq) + CO32–(aq), so Ksp = [Ca2+][CO3

2–]. If all the carbonate has dissociated, then [Ca2+] = [CO3

2–] and we can write Ksp = [Ca2+][Ca2+] = [Ca2+]2. Thus, [Ca2+] = (Ksp)1/2 = (4.8 × 10–9)1/2 M.

31. D. Equilibrium constants, such as Ksp, can be changed only by a change in temperature, so we eliminate choices A and B. Thus, the question becomes whether [CO3

2–] increases or decreases with the addition of excess Ca2+. Since the question tells us that CaCO3 precipitates, the ions in solution must be combining to form the solid, thereby lowering the concentration of CO3

2–.

Passage VI (Questions 32–36)

32. D. The frequency of a simple pendulum is given by the formula f = (1/2π)(g/L)1/2. Since angular frequency ω (in radians per second) is related to frequency f (in hertz) by the equation ω = 2πf, the angular frequency of a simple pendulum is given by ω = (g/L)1/2. Note that choices A and B can be eliminated because the units in both these choices are incorrect: mg/L has units of N/m or kg·s–2, and (L/g)1/2 has units of seconds.

33. C. Any angle in degrees can be converted to radians by multiplying by π/180°. So, an angle of 30° is equivalent to an angle of 30° × π/180° (which is π/6) radians.

34. B. In Figure 3, Resistors R2 and R1 are in series, so they are equivalent to a single 200 Ω + 200 Ω = 400 Ω resistor (which weʼll call R21). Resistors R3 and RSG are also in series and equivalent to a single 400 Ω resistor, which weʼll call R3SG. These two equivalent resistors, R21 and R3SG, are in parallel, so the overall equivalent resistance, Req, is (400 · 400)/(400 + 400) = 200 Ω.Since the battery has a voltage of 12 V and the overall resistance of the circuit is 200 Ω, the current from the battery must be I = V/Req = (12 V)/(200 Ω) = (60/1000) A = 60 mA.

35. B. The restoring force is the component of the bobʼs weight, mg, that is directed toward the vertical equilibrium position. As the following figure shows, this component is mg sin .

Note: The second diagram in Figure 1 in the passage also illustrates this component.

Page 7: AAMC MCAT Test 8 Solnsdocshare01.docshare.tips/files/23478/234782350.pdf · 2016. 12. 13. · MCAT Practice Test 8 Solutions 4 this fact from nuclear physics, you can still choose

Physical Sciences Solutions 7

36. D. At the molecular level, we can think of the atoms in the string as if they were held together by tiny springs, which can be stretched or compressed, and thus determine the tension in the string. Therefore, choice D is best.

Passage VII (Questions 37–43)

37. A. The buoyant force acting on an object is equal to the weight of the fluid it displaces. Since each lead weight is completely submerged, the volume of water it displaces is equal to its full volume, V = 4 × 10–6 m3. The weight of a volume V of water is equal to ρH2OVg, so in this case, we find that the buoyant force is ρH2OVg = (103 kg/m3)(4 × 10–6 m3)(10 m/s2) = 4 × 10–2 N.

38. D. The results shown in Figure 1 indicate how the temperature of the air inside the balloon varies with the balloonʼs depth. Since “the temperature of the water is kept uniform throughout the tank,” knowing the total mass of the water in the tank is irrelevant.

39. B. As the balloon rises, its depth decreases, and Figure 1 in the passage shows that as the depth decreases, the temperature also decreases. (You may find it helpful to place your finger on the curve in Figure 1 at its right-hand end, and move your finger along the curve to the left. This movement to the left corresponds to the depth decreasing—that is, the balloon rising. Note that the temperature decreases.) This immediately eliminates choice C, which says the temperature of the gas increases. Also, as the balloon rises in the tank, the water pressure on it decreases, so the gas in the balloon is not being compressed as it rises, it is expanding; this eliminates choice D. Now, since the temperature of a gas is directly related to the average kinetic energy of its molecules, the fact that the temperature decreases immediately implies that the internal energy of the gas decreases also. Choice B includes both the expansion of the gas and the resulting decrease in its internal energy, and is the best answer.

40. A. The change in temperature of the gas inside the balloon is a direct result of the work done by the gas expanding the balloon. If the balloon were sufficiently inflexible, then any expansion—and resulting temperature drop—would be far more difficult to detect.

41. C. The weight of the balloon (not to mention the weight of the piece of lead thatʼs attached to it) is always acting on the balloon; this eliminates choices B and D, which do not include weight. Since the balloon is submerged in water, buoyancy also acts on it. Thus, the answer must be C.

42. D. The Continuity Equation tells us that flow speed is greatest where the cross-sectional area of the flow tube is smallest. The tank itself has an inside diameter of 5 meters (which implies a cross-sectional area of nearly 20 m2!), but the drain pipe has a cross-sectional area of only 0.2 m2 next to the tank, which tapers to just 0.1 m2 at the other end. Since the tapered end has the smallest cross-sectional area, the flow speed here should be the greatest.

43. B. If we consider the air inside the balloon as an ideal gas, then we can apply the Ideal-Gas law, PV = nRT. Since n and T remain constant here, P is inversely proportional to V. So, if V doubles, then P is reduced to one-half its original value.

Passage VIII (Questions 44–48)

44. D. First, we can eliminate choices B and C: oxidation is a loss of electrons, and reduction is a gain. Since it is reduction that occurs at the cathode, the answer is D.

45. C. Since calcium is an alkaline earth metal in the second column of the periodic table, it will form a +2 ion to achieve a noble-gas configuration. Ca2+ has 20 – 2 = 18 electrons, and only choice C accounts for 18 electrons.

Page 8: AAMC MCAT Test 8 Solnsdocshare01.docshare.tips/files/23478/234782350.pdf · 2016. 12. 13. · MCAT Practice Test 8 Solutions 4 this fact from nuclear physics, you can still choose

MCAT Practice Test 8 Solutions8

46. B. The reactions described in Table 1 are displacement/redox reactions between Group I or Group II metals and water. The intensity of the resulting reaction is directly related to how easily the metal atoms can be oxidized—that is, to how easily electrons can be removed from them. Choice A can be eliminated since electronegativity refers to the strength with which an atom holds on to shared electrons in a bond, and choice C can be eliminated because metals form cations, while electron affinity refers to the energy associated with forming an anion. Polarizability (choice D) refers to the ability to create a dipole within a molecule or ion as the electron cloud becomes unevenly distributed; it is not related to the loss of an electron from the reacting species, as ionization potential is. The best answer is B.

47. A. The reaction of Ca(s) with water is Ca + 2 H2O → Ca2+ + 2 OH– + H2. Because hydrogen gas is produced, we eliminate choices C and D. To determine the resulting volume of hydrogen gas, we first note that according to the given balanced reaction, for each mole of Ca consumed, one mole of H2 is produced. Since 0.40 g of calcium is equal to 0.01 mole (because the molar mass of calcium is 40 g/mol), this reaction will produce 0.01 mole of H2(g). Now, since the given conditions are very close to STP, we can say that 1 mole of gas will occupy approximately 22.4 L. Therefore, 0.01 mole of H2(g) will occupy approximately 0.224 L = 224 mL, so choice A is best. Note: You could calculate a more precise value of the volume using the Ideal-Gas law in the form V = nRT/P: with n = 0.01 mol, R = 0.0821 L·atm/mol·K, T = 27°C = 300 K, and P = 1 atm, you would find that

V = nRTP

=(0.01 mol)(0.0821 L ⋅atm

mol⋅K )(300 K)1 atm

= (0.0821)(3) ≈ 0.25 L = 250 mL

48. A. Because each of the metals described in the question produces a different color when heated, it follows that each emits light at specific wavelengths. An element emits light when electrons in higher energy levels ʻdrop ̓ to lower energy levels. Since the energy levels are quantized, the photon energies emitted by these transitions are also quantized, so light of only specific wavelengths is produced. Choice A states this. Choices B and D describe absorption of light of specific wavelengths (not emission, which is the phenomenon weʼre trying to account for here), and choice C says that electrons are converted into gases, which is false.

Free-Standing Questions (49 through 53)

49. D. Since Point A is in the liquid phase and Point B is in the gas phase, the arrow from A to B represents a liquid-to-gas phase change. This is vaporization.

50. D. Choice D describes exactly what makes a material a good insulator/poor conductor: its electrons cannot move easily from one atom to another.

51. D. Starting with the formula q = mcΔT, we solve for c and get c = q/(m·ΔT). If q = 250 J, m = 0.1 kg, and ΔT = 25°C – 20°C = 5°C, then c = q/(m·ΔT) = (250 J)/(0.1 kg · 5°C) = 500 J/(kg·°C).

52. B. The resistance of a wire is determined by the formula R = ρL/A, where ρ is its resistivity, L its length, and A its cross-sectional area. These are exactly the quantities listed in choice B.

53. B. “Most stable” means “lowest energy”. The electron configuration of a nitrogen atom in its ground state is 1s2 2s2 2p3, choice B. All the other choices give the configuration of a nitrogen atom in which an electron in a lower-energy orbital has been promoted to a higher-energy orbital (2s & 2p in choice A, 1s & 2p in C, and 2p & 3s in D). Each of these three configurations thus describes a nitrogen atom with more energy (and thus lower stability) than the one described in choice B.

Page 9: AAMC MCAT Test 8 Solnsdocshare01.docshare.tips/files/23478/234782350.pdf · 2016. 12. 13. · MCAT Practice Test 8 Solutions 4 this fact from nuclear physics, you can still choose

Physical Sciences Solutions 9

Passage IX (Questions 54–58)

54. B. The passage tells us that 7.15 g of Na2CO3•10H2O was dissolved in the initial 50 mL solution. Since the molar mass of Na2CO3•10H2O is given to be 286.14, we can determine that the amount of Na2CO3•10H2O dissolved was

7.15 g286.14 g mol

≈ 7280

= 140

mol

Because each formula unit yields 2 Na+ ions, there were 2( 140) = 1

20 mol of sodium ions in solution. Finally, we use the definition “1 mole ≈ 6 × 1023 particles” to conclude that the number of sodium ions in solution was 1

20 (6 × 1023) = 3 × 1022.

55. C. Sodium ion, Na+, is neutral in solution, and carbonate ion, CO32–, is a polyprotic base. Therefore, a solution of Na2CO3

is basic and thus turns red litmus to blue. Since the reaction between carbonate and water is CO32– + H2O → HCO3

– + OH–, choice C is best.

56. A. The passage tells us that 6.57 g of NiSO4•6H2O was dissolved in the initial 50 mL solution. Since the molar mass of NiSO4•6H2O is given to be 262.84, we can determine that the amount of NiSO4•6H2O dissolved was

6.57 g262.84 g mol

= 6.57262.84

× 22

≈ 13520

= 140

mol

Therefore, there is 140 mol of Ni2+ in the solution. Also, from our calculation in Question 54 above, there is also 1

40 mol of CO3

2– in the solution. Thus, the maximum amount of NiCO3 that could form as precipitate is 140 mol = 0.025 mol.

57. C. The carbonate ions in solution react with the strong acid to produce carbon dioxide: CO32– + 2 H+ CO2(g) + H2O.

58. C. Electron transitions (caused by the selective absorption of energy from visible light) between d-orbitals in transition-metal complexes give rise to color. In this case, the nickel(II) sulfate solution absorbs red light, so the solution appears green (the complementary color of red). Thus, choice C is the answer. The other choices are irrelevant to the production of color.

Passage X (Questions 59–65)

59. A. The passage tells us that “when a photonʼs E exceeds W [the work function], the photon ejects an electron from the metal. The kinetic energy, K, of the ejected electron is the difference between E and W.” Since the work function of Metal L is 8.0 eV but the photon has an energy of only 7.0 eV, the photonʼs E does not exceed W, so no electron will be ejected from Metal L, which we can consider here to be an electron with no kinetic energy. Only choice A lists “0.0” for Metal L.

60. B. In order to increase the current I, we would want to increase the number of electrons ejected from the metal plate reaching the collector plate (marked “B” in Figure 2 of the passage) per unit time. If we were to replace the metal plate with one having a higher work function (choice A), there will either be no electrons ejected from the plate (if the higher work function is greater than the energy of the incoming photons) or, if electrons are still ejected, their kinetic energy will be decreased, thus decreasing the number that could reach the collector plate per second. If the number of photons having energies less than W were increased (choice C), there will certainly be an effect on the number of electrons ejected, because none of these low-energy photons could cause the ejection of any electrons. Finally, if the average frequency of the photons were decreased (choice D), their average energy would be decreased also (because photon energy E is directly proportional to photon frequency f, through the equation E = hf ), and the situation would be the same as the one described earlier for choice A. The answer must be B. If Plate B were made more positive relative to Plate A, then the negatively charged electrons would be attracted more strongly to Plate B, thus increasing the current.

61. D. First, we can eliminate choices A and B. If the wavelength is increased, then the energy of the light is reduced (because E is inversely proportional to λ, since E = hc/λ). Thus, the fringes would not become brighter or bluer. The passage tells us that “the brightest fringes occur where wave maxima overlap with other maxima or where minima overlap with other minima.” If λ is increased, then the maxima and minima move farther apart, so the fringes will move farther apart, too.

Page 10: AAMC MCAT Test 8 Solnsdocshare01.docshare.tips/files/23478/234782350.pdf · 2016. 12. 13. · MCAT Practice Test 8 Solutions 4 this fact from nuclear physics, you can still choose

MCAT Practice Test 8 Solutions10

62. C. The passage tells us what causes the fringes to appear on the screen: the overlapping of the two beams from S1 and S2. In other words, it is the interference of the two beams that creates the fringes; in fact, the passage even calls them ʻinterference fringesʼ. So, if S1 were covered (choice C), there would be only one beam (the one from S2) reaching the screen, so there would be no interference, and thus no interference fringes. (Diffractive effects are ignored here.) Choice A, B, or D could change the pattern—either causing the fringes to move farther apart (choice A or D) or increasing their contrast (choice B)—but none of these would cause the fringes to disappear.

63. A. As a spherical wave spreads out, its wave front becomes flatter. (A portion of a large circle looks straighter than an equal-angle portion of a small circle.) So, the greater the distance the spherical waves leaving slit S can travel, the flatter the wave fronts will be. Referring to Figure 1, this means that the light entering slits S1 and S2 will appear most like a plane wave if the distance between T1 and T2 is large.

64. B. Since Wave A is 180° out of phase with Wave B, when Wave A meets Wave B, they will cancel each other completely. This leaves just the amplitude of Wave C, which is E.

65. D. First, eliminate choice A: The work function of a metal is an intrinsic characteristic of the metal and is not affected by the light we choose to shine on it. Choice B is also wrong: The intensity of a beam of light depends on the number of photons, not on their frequency (or energy)—for example, a beam of red light will be more intense than a beam of blue light if the red beam contains more photons. We also reject choice C, because evidently light waves can carry sufficient energy to eject an electron, since the photoelectric effect is a real and common phenomenon. The answer is D. If a single photon, which is the least intense any actual “beam” of light can be, can cause the ejection of an electron, then this would contradict the wave theory that says only light “of sufficient intensity must be used.”

Passage XI (Questions 66–71)

66. C. The passage tells us that the ionic compounds [one of which was contained in Solution A] were completely soluble in water. The choices for this question match the anions listed in Table 1, and of these, only F– gave no precipitate with Ag+.

67. D. The passage is not needed to answer this question. When the reaction quotient Q for the dissolution of a compound into its ionic components is greater than the compoundʼs tabulated solubility product (its Ksp value), a precipitate forms.

68. A. A reaction in which cations and anions in solution ʻexchange partners ̓and form a precipitate is an example of a meta-thesis reaction (choice A). Alternatively, you can eliminate the other three choices, none of which applies to the precipitation reactions in Table 1.

69. B. From Table 1, we see that while the cation Ag+ caused the formation of three precipitates, and Cu2+ and Fe3+ each formed two, Ca2+ caused the formation of only one precipitate. Therefore, the cation that allowed for the greatest number of soluble compounds was Ca2+.

70. A. Because BaSO4 has the lowest Ksp value, it will form the fewest Ba2+ ions in aqueous solution, and thus be the least toxic of the four choices.

71. B. A solution of FeCl3 contains Fe3+ and Cl– ions, and a solution of FeF3 contains Fe3+ and F– ions. According to Table 1, the addition of Ca2+ (choice B) would react with F–, causing a white precipitate to form in the FeF3 solution, but not react with Cl– in the FeCl3 solution. This would certainly allow a researcher to distinguish the two solutions, so choice B is the answer. Adding CrO4

2– or S2– (choice A or C) would, according to the information provided in Table 1, cause brown precipitate to form in both solutions, while adding Cu2+ (choice D) would cause no reaction in either solution. Therefore, none of these would be useful in differentiating between the solutions.

Page 11: AAMC MCAT Test 8 Solnsdocshare01.docshare.tips/files/23478/234782350.pdf · 2016. 12. 13. · MCAT Practice Test 8 Solutions 4 this fact from nuclear physics, you can still choose

Physical Sciences Solutions 11

Free-Standing Questions (72 through 77)

72. B. The terminal voltage of the battery is its full electromotive force � minus the voltage drop across its internal resistance. Since the current is i and the internal resistance is r, the voltage drop across the internal resistance is ir, so the terminal voltage of the battery is reduced to � – ir.

73. B. Because the medium has a higher refractive index, n, than that of the air, the beam will bend toward the normal as it enters the medium from the air. Therefore, θ > α, which eliminates choices A and C. Since the angle of incidence for the beam leaving the medium is α, the angle of refraction here must be θ.

θ

α α α α

θair

medium(n)

sin θ = n sin α n sin α = sin θ

angle ofincidence

angle ofrefraction

angle ofincidence

angle ofrefraction

Law of Reflection

(That is, the sizes of the angles made with the normal when the beam goes from medium-to-air are the same as when the beam went from air-to-medium. Only the terms “incidence” and “refraction” would be interchanged.) Thus, θʹ = θ.

74. D. We can eliminate choice A since it would be statistically impossible for the molecules of one gas to transfer their excess kinetic energy to the molecules of another gas if the two gases are in equilibrium. Choice B is wrong since in the kinetic theory of gases, collisions are elastic, so a molecule colliding with a wall of the container will have the same kinetic energy after the collision as before. Choice C is wrong because we know from the question that the molecules in the two phases have the same average kinetic energy; choice C claims that the vapor molecules have a higher kinetic energy. Alternatively, the potential energy of a substance increases as the phase changes from liquid to gas. Thus, choice C can be eliminated because it says that the vapor has a lower potential energy. The answer must be D. Molecules that escape the surface of the liquid must expend energy to overcome the intermolecular forces that held them to the liquid, so the higher kinetic energy these molecules had before vaporizing is reduced by this expenditure.

75. A. The speed of a wave is determined by the medium it travels through, not by the waveʼs intensity, frequency, or wavelength. Since the wave is traveling through the same medium (namely, air) both before and after reflection, its speed will remain the same, too; the answer must be A. Choices C and D could be eliminated because of the Doppler Effect: Since the reflector is moving toward the source, we know that the wave after reflection will have a higher frequency and a shorter wavelength than the wave before reflection. As for choice B, intensity, the fact that the question describes the reflector as ʻimperfect ̓tells us that the intensity after reflection will be less than before.

76. D. The formula for the Doppler shift, f ʹ = f (v ± vD)/(v ± vS), involves only the speed of sound in the medium (v), the speeds of the source and observer (vS and vD), and the emitted and observed frequencies ( f and f ʹ). It does not involve the distance between source and observer.

77. D. The impulse–momentum theorem tells us that the change in the momentum of an object is equal to the impulse applied to it, so the answer is D. Also, since the units of a change in momentum are the same as the units of momentum itself (namely, kg·m/s), a quantity thatʼs equal to a change in momentum must have these units, too. But none of the quantities in A, B, or C has these units, thus eliminating all three of these choices. (Choice A is a trap. An alternative form of Newtonʼs Second law, F = Δp/Δt, says that force is equal to the rate of change of momentum, not merely a change in momentum.)

Page 12: AAMC MCAT Test 8 Solnsdocshare01.docshare.tips/files/23478/234782350.pdf · 2016. 12. 13. · MCAT Practice Test 8 Solutions 4 this fact from nuclear physics, you can still choose

MCAT Practice Test 8 Solutions12

VERBAL REASONING SOLUTIONS

Passage I (Questions 78–83)

78. CA: First and most importantly, the passage never suggests that the U.S. has a low literacy rate—this is outside knowledge.

Second, even if the author did tell us that the U.S. had a low literacy rate, that would not prove that the introduction of an alphabet changes cognitive habits.

B: There is no research or data cited in the passage to support this claim; it is simply asserted in lines 56–57.C: Yes. There is no reason, based on the passage, to believe the assertion to be false; therefore, it perhaps is true.

However, there is no evidence or reasoning backing up the assertion; it is stated in lines 56–57 with no support. The other items on the list (social relations, notions of community, history, and religion [lines 57–59]) donʼt constitute separate evidence showing that introducing an alphabet changes cognitive habits.

D: While the author does assert that television viewing is pervasive (lines 27–29), this is not inconsistent with the claim that an alphabet changes cognitive habits. Nothing in the passage indicates that the changes brought about by introduction of an alphabet are mutually exclusive with watching lots of television.

79. ANote: The correct answer will be the choice that is most inconsistent with the claim that “By ushering in the Age of Television, America has given the world the clearest available glimpse of the Huxleyan future” (lines 29–31). Part of this claim is the argument that people are unaware of the changes brought about by television in particular and technology in general; that is, “they did not know what they were laughing about and why they had stopped thinking” (lines 74–75), because the Huxleyan world is difficult to recognize (lines 35–38).A: Yes. This choice weakens the authorʼs argument by indicating that people are in fact aware of how television and

the media affect their lives, and that they can consciously evaluate the content and quality of what they see and hear. That is, they are able to understand “the politics and epistemology of the media” (lines 70–71).

B: This choice suggests neither that television has no impact (if anything, it suggests that it may have an effect) nor that people are aware of its effects. This choice is consistent, not inconsistent, with the authorʼs suggestion that “people [have] become an audience and their public business [has become] a vaudeville act” (lines 18–19); we watch and are perhaps entertained, but we donʼt intelligently participate.

C: This choice, like Choice B, is not inconsistent with the authorʼs description of a Huxleyan world as one in which we watch “public business” as a form of entertainment (line 19) without really understanding what we are watching and why (lines 71–75).

D: This choice has no direct relevance to the authorʼs argument in the passage. The authorʼs claim that Huxleyʼs Brave New World describes our society better than Orwellʼs 1984 (lines 22–24) does not rest on a claim that more people have read Huxleyʼs work.

80. CNote: Notice the similarity between choices A, B, and D; all three indicate that people would accept and respond to Huxleyʼs warning. This is inconsistent with the authorʼs argument in lines 32–44. In this case, the correct answer is the one that is not like the others.A: The author states that those who voice Huxleyʼs warning must do so “at a near hysterical pitch…because what they

want others to see appears benign, when it is not invisible altogether” (lines 32–37). (This is in contrast to an Orwellian or prison-like world, which is “much easier to recognize, and to oppose” [lines 37–38].) Yet because they must “raise their voices” to such an extreme level in order to be heard, they may be written off as “everything from wimps to public nuisances to Jeremiahs” (lines 32–35). The suggestion that people would agree with the warning and act accordingly is inconsistent with these statements in paragraph 4.

B: See the explanation for Choice A. The passage suggests that people are more likely to ignore or discount such a warning.C: Yes. The author states that because the aspects of a Huxleyan world are so difficult to recognize and resist (lines

35–38), people “who speak about this matter must often raise their voices to a near-hysterical pitch, inviting the charge that they are everything from wimps to public nuisances to Jeremiahs” (lines 32–35). That is, they will likely be written off as incorrect and/or too extreme.

D: See the explanation for Choice A. The author suggests instead that people are unlikely to be receptive, in part because the problematic aspect of Huxleyʼs world “appears benign, when it is not invisible altogether” (lines 36–37).

Page 13: AAMC MCAT Test 8 Solnsdocshare01.docshare.tips/files/23478/234782350.pdf · 2016. 12. 13. · MCAT Practice Test 8 Solutions 4 this fact from nuclear physics, you can still choose

13Verbal Reasoning Solutions

81. DA: The author suggests that people are unaware that public culture has become trivialized (lines 18–21 and 71–75), not that

they actively prefer what is most trivial. If the public were able to distinguish between which candidates were most and least trivial, it would be inconsistent with the authorʼs suggestion “that they did not know what they were laughing about and why they had stopped thinking” (lines 74–75).

B: As in Choice A, this answer suggests a level of awareness on the part of the public that is inconsistent with the authorʼs description. Furthermore, this choice suggests that the public would resist the trivialization of public life. However, the author argues that these changes, unlike the creation of an Orwellian world, are likely either to go unnoticed or to appear benign, and so are much harder to resist and oppose (lines 37–38).

C: See the explanation for Choice B. The author suggests that the public would not even notice that the candidates and discussion were trivial, and that if this were noticed it would likely not be seen as a problem (lines 35–44). While an oppressive Orwellian world is easy to recognize and resist, people are unaware that the changes described by Huxley are occurring (lines 35–38) or that they are potentially disastrous (lines 20–21 and 68–69).

D: Yes. The author describes Huxleyʼs world as one in which “public business becomes a vaudeville act” (line 19) and “serious discourse dissolves into giggles” (lines 42–43). A campaign reduced to trivialities would fit this description. The author also argues that although these changes brought about by television can lead to disaster (lines 20–21 and 68-69), the public either doesnʼt perceive that things have changed or doesnʼt perceive the changes as dangerous (lines 35–38). Therefore, it is likely that the public would not notice the triviality of the campaign.

82. DItem I: False. The author suggests just the opposite. The passage indicates that because of television, “public business”

has become “a vaudeville act” (line 19), “serious discourse dissolves into giggles” (lines 42–43) and our culture is “being drained by laughter” (lines 43–44).

Item II: True. In paragraph 3 the author identifies technology, but more specifically television, as the causal factor in the creation of a Huxleyan world (lines 27–31). The author goes on to say that television “imposes a way of life, a set of relations among people and ideas” (lines 47–48). The passage also draws an analogy between an alphabet, movable type, and speed-of-light transmission of images such that when introduced into a society, all three “change its cognitive habits, its social relations, its notions of community, history, and religion,” and that speed-of-light transmission of images (television) does so to an extreme degree (lines 56–61).

Item III: True. The author argues that technology is not “neutral,” and that “to make the assumption that technology is always a friend to culture is…stupidity plain and simple” (lines 54–56). The author also argues that the Age of Television (technology) has brought us close to realizing a “Huxleyan future” (lines 29–31) and that “[Huxley] believed…that we are in a race between education and disaster” (lines 68–69). The purpose of the passage as a whole is to warn us that Huxleyʼs vision of “spiritual devastation” (lines 13–14) and “culture death” (line 20) is in danger of being made real through the influence of technology, in particular television.

83. AA: Yes. If the presidential election were devoid of serious discourse (line 42) and consisted merely of a personality

contest, this would support the authorʼs claim that we are approaching a Huxleyan world in which “public business” has become “a vaudeville act” (line 19), functioning only as a source of entertainment for an unthinking population (line 75).

B: This choice (“no one”) is too extreme. The passage describes “those who speak about this matter” (lines 32–37); some people are in fact warning the public about a possible “Huxleyan future” (lines 30–31).

C: The contestants in a personality contest do not watch the audience; the audience (the people) watch the politicians (“Big Brother”) in a Huxleyan world [lines 16–18]). This study offers no support for Orwellʼs prophecies.

D: This is the right answer to the wrong question. The author does refer to a belief in “the inevitability of progress” (lines 66–67) as contributing to the power of the ideology of technology. However, the study cited in the question stem does not offer evidence that people hold this belief.

Page 14: AAMC MCAT Test 8 Solnsdocshare01.docshare.tips/files/23478/234782350.pdf · 2016. 12. 13. · MCAT Practice Test 8 Solutions 4 this fact from nuclear physics, you can still choose

MCAT Practice Test 8 Solutions14

Passage II (Questions 84–93)

84. CA: This choice is inconsistent with the authorʼs statement that “useful as the experience of literary creation is, the task of

the student is completely distinct. The student must translate the experience of literature into intellectual terms…” (lines 11–14). This statement is intended to refute the argument, described in the first paragraph, that “one cannot understand literature unless one writes it” (lines 6–7).

B: This choice is too narrow to be the main idea. While the author would agree that it is true (lines 42–48), the point of the passage as a whole is to argue that the methodologies of literary study are to a significant degree distinct.

C: Yes. The author describes similarities and overlap between the two (paragraph 3) in order to go on to make the central argument that literature cannot be studied only through using scientific methodologies (lines 40–50 and 60–63).

D: This choice is too narrow to be the main idea. The author makes this claim in the last paragraph (lines 53–60) only in service of the larger point that “the achievements [of the humanistic disciplines] are nevertheless real and permanent” (lines 56–59), and that they represent “valid methods of knowing” (lines 53–56).

85. AItem I: True. The author states that “The student must translate the experience of literature into intellectual terms”

(lines 12–14), and that “The problem is one of how intellectually to deal with art, and with literary art specifically” (lines 21–22).

Item II: False. The passage argues that literary study must find a way of dealing with irrational or unrational subject matter in a rational way (lines 12–17) but does not suggest that this requires sorting out the rational from the irrational elements within a literary work.

Item III: False. In the first paragraph the author argues that literature as art and literary study are two distinct enterprises; “One is creative, an art; the other, if not precisely a science, is a species of knowledge or of learning” (lines 1–4). While “it has been argued that one cannot understand literature unless one writes it” (lines 6–7), the author disagrees with this claim that the two must be integrated. Finally, the idea that one must study literature in order to create literature is never mentioned in the passage.

86. CA: The author mentions art history (“the historian of painting”) and musicology in line 19 in order to draw an analogy with

literary study. The point of the analogy is that in all of these fields it is necessary to study sometimes irrational or unrational subject matter in a rational, intellectual way (lines 12–20). Later in the passage, the author argues that strict scientific methodology is insufficient for the study of literature (lines 40–42, 47–50, and 60–63). Thus, the passage indicates that art history and musicology, like literature, should not be approached with a “strict scientific methodology.” This choice contradicts the passage.

B: The passage argues that one does not need to be a writer in order to study literature (lines 1–12). Since the author draws an analogy between literary study on one hand, and art history and musicology on the other (lines 17–19), we can infer that one could be an art historian or musicologist without practicing the art form itself. This choice contradicts the passage.

C: Yes. In the second paragraph, the author first states that a student of literature must “translate the experience of literature into intellectual terms, assimilate it to a coherent scheme which must be rational if it is to be knowledge” (lines 12–15). This requires dealing rationally with irrational or unrational subject matter, and in this way “the student of literature will not be therefore in any other position than the historian of painting or the musicologist” (lines 17–19). In the beginning of the third paragraph, the author states that “The problem is one of how intellectually to deal with art, and with literary art specifically.” Thus the author draws an analogy between literary study, art history, and musicology, and suggests that all three must be “approached intellectually, despite their irrational components.”

D: This choice contradicts the passage. The author draws an analogy between literary study, art history, and musicology on the basis that they all require dealing with sometimes irrational or unrational elements in a coherent, rational, and intellectual way (lines 12–23). See also the explanation for Choice C, and note that Choice C and Choice D are essentially opposites of each other.

Page 15: AAMC MCAT Test 8 Solnsdocshare01.docshare.tips/files/23478/234782350.pdf · 2016. 12. 13. · MCAT Practice Test 8 Solutions 4 this fact from nuclear physics, you can still choose

15Verbal Reasoning Solutions

87. DA: This would involve applying the second of the three ways described by the author of transferring the methods of the

natural sciences to the study of literature. That is, it would entail “[imitating] the methods of natural science through the study of causal antecedents and origins…. Scientific causality is used to explain literary phenomena by the assignment of determining causes to economic, social, and political conditions” (lines 30–35). However, the author goes on to argue that the transfer of natural science methodology to the study of literature “has not fulfilled the expectations with which it was made originally” (lines 41–42). Therefore, the author would not identify this choice as the most important task to be performed.

B: As in Choices A and C, this choice describes applying methods transferred from the natural sciences, a transfer which the author indicates is not fully adequate to the purposes of literary study (lines 40–42). This choice involves “the introduction of the quantitative methods appropriately used in some sciences—i.e., statistics, charts, and graphs” (lines 35–37), in this case statistics.

C: As in Choices A and B, this choice describes the application of methods transferred from the natural science; the author states that “this transfer has not fulfilled the expectations with which it was made originally” (lines 41–42). By tracing prior influences and the influence of the story on later works, the scholar would be applying the third scientific method described in paragraph 2, that is, “the attempt to use biological concepts in the tracing of the evolution of literature” (lines 38–39).

D: Yes. First, this is the only choice that is not tainted by some association with the scientific methods discredited by the author of the passage (paragraph 2 and lines 40–43). Second, by isolating the story elements that explain the storyʼs popularity, the scholar would be “translat[ing] the experience of literature into intellectual terms” (the experience of the story would relate to its popularity) and applying to it a coherent and rational scheme (lines 12–15) and analysis (line 45) by isolating certain story elements.

88. AA: Yes. This choice paraphrases the passage in lines 42–47. In this section, the author is listing the methods which are

“common to all types of systematic knowledge,” including literary study.B: While these methods (common to both science and literary study) are described as “appropriate” in lines 42–47, the

second part of this choice is incorrect. The author argues that the methods of science, including objectivity and quantitative methods (lines 27–42) are insufficient, and that “the other solution commends itself: Literary scholarship has its own valid methods, which are not always those of the natural sciences” (lines 47–49).

C: First, the author describes these methods as appropriate (while not sufficient by themselves) for the study of literature (lines 42–47). Second, while the author does suggest that literature includes irrational aspects, the author argues that students of literature are therefore called upon to find a rational, coherent, and systematic scheme by which irrationality can be dealt with intellectually (lines 12–22).

D: While the author would agree that the humanities and the natural sciences have, to some degree, different aims and purposes, induction and deduction etc. are listed as methods common to both science and literary study (lines 42–47). Therefore, their use in the study of literature would be appropriate.

89. DA: The author argues that creating literature and studying it are “two distinct activities” (lines 1–2). While investigating the

process of literary creation could potentially provide support for this distinction, the passage as a whole is about the study of literature and how this study requires methods that are in part different than scientific methods. Therefore, the kind of investigation described in this answer choice would not provide “the most convincing evidence for passage assertions.”

B: This choice is inconsistent with the authorʼs assertions that “Literary scholarship has its own valid methods, which are not always those of the natural sciences but are nevertheless intellectual methods” (lines 48–50), and that “there is this difference between the methods and aims of the natural sciences and the humanities” (lines 61–63). See also the explanation for Choice C.

C: See the explanation for Choice B. The author asserts that, while there is some overlap (lines 42–47), there are methods to literary scholarship that are distinct from scientific methodology. Applying a particular scientific technique to a literary work as well as to a scientific problem would undermine, not support this assertion.

D: Yes. The author argues that “Literary scholarship has its own valid methods, which are not always those of the natural sciences but are nevertheless intellectual methods” (lines 48–50). This is the main idea of the passage as a whole (see also paragraph 5). By comparing a literary to a scientific analysis of a literary work, the author could demonstrate the differences between the two analyses. This comparison could also demonstrate what might be learned from a literary analysis that cannot be learned from a purely scientific approach. Thus, out of these four answer choices, choice D could provide the most convincing evidence in support of the passageʼs main assertions.

Page 16: AAMC MCAT Test 8 Solnsdocshare01.docshare.tips/files/23478/234782350.pdf · 2016. 12. 13. · MCAT Practice Test 8 Solutions 4 this fact from nuclear physics, you can still choose

MCAT Practice Test 8 Solutions16

90. BA: While defining these terms might clarify the meanings of the terms themselves, it would not go very far (and not as far as

Choice B) in clarifying what they mean in the specific context of the statement cited in the question stem.B: Yes. An example of an unrational statement in a literary work would clarify not only what the author means

by “unrational”, but also what the author means by saying that literature itself contains “unrational elements.” Furthermore, given that “unrational” is (to most people) a less familiar word than “irrational”, an example of what the author means by “unrational” would go furthest, of these four choices, in clarifying the authorʼs meaning. In general, the best way (on the MCAT) of clarifying (or strengthening) a statement is to give a specific example in which that statement is true.

C: The irrationality of the process, and the irrationality or unrationality of certain existing elements in a literary work, are two separate issues. While an irrational process may lead to the creation of irrational elements, this answer choice would at the most partially explain why irrational elements were created, not how the elements are irrational or unrational (or what the author means by “unrational elements”).

D: As in choice C, this statement might help explain why literature contains irrational elements, but not what the author means by “unrational elements” (that is, how these elements are in fact unrational).

91. DA: This choice is inconsistent with the main idea of the passage. While the author does state that there is a degree of overlap

between scientific methodology and methodology appropriate to other forms of “systematic knowledge” (lines 44–47), including literary study, the passage as a whole is intended to argue that the study of literature has its own unique methods (lines 40–42, 47–50, and 51–63). The unique character of literary study, not what it has in common with the natural sciences, is what makes it important.

B: The author argues that creating literature and studying literature are “distinct activities: One is creative, an art; the other [literary study], if not precisely a science, is a species of knowledge or of learning” (lines 2–4). The author never suggests that the study of literature leads to increased creativity on the part of the student.

C: This is the right answer to the wrong question. That is, the passage teaches us that truth (“truth” is mentioned in line 51, with the implication that truth and knowledge are related) can be discovered through non-scientific means (lines 48–50 and 60–63), but the author does not suggest that the study of literature is important because literary study itself teaches us about non-scientific paths to the truth. That is, literary study is important because it offers us this knowledge or truth, not because it demonstrates the availability of such forms of truth. Compare this choice to Choice D to see the subtle distinction between the two: “teaches that” in Choice C versus “offers” in choice D.

D: Yes. The final paragraph argues that the humanities “had worked out valid methods of knowing” before the existence of the modern scientific method (lines 53–56), and that the humanities, including literary study, offers its own forms of knowledge and truth which cannot be achieved only through the scientific method (lines 48–54 and 60–63). Furthermore, in the first paragraph the author describes literary study as “a species of knowledge or of learning” (lines 2–4). This unique contribution is, according to the passage, what makes the study of literature important; this is, in fact, the main idea of the passage. Compare this choice to Choice C: in C, literary study is important because it teaches us about the existence of other (non-scientific) paths to truth or knowledge, while in Choice D literary study is important because it is that path.

92. BItem I: False. The author lists analysis as one method which is “common to all types of systematic knowledge” (lines

42–47), including both literary study and science. The success of systematic analysis of text would be an illustration of the success, not the failure, of at least one method used by the natural sciences.

Item II: True. In lines 42–47, the author lists ways in which the natural sciences and literary study overlap. “Analysis” is one method “common to all types of systematic knowledge.” Therefore, a successful “systematic analysis of text” would be evidence for this overlap.

Item III: False. The fact that some methods used by the sciences (in this case, systematic analysis) can also be successfully used to study literature (as stated by the passage itself in lines 42–47) is not inconsistent with the claim that other fields of study have worked out their own “valid methods of knowing” (lines 53–56). The author of the passage argues that there is some overlap, but that non-scientific fields also have some of their own unique methods of study.

Page 17: AAMC MCAT Test 8 Solnsdocshare01.docshare.tips/files/23478/234782350.pdf · 2016. 12. 13. · MCAT Practice Test 8 Solutions 4 this fact from nuclear physics, you can still choose

17Verbal Reasoning Solutions

93. BNote: When the question stem states that “Elsewhere, the author says…,” this means that the quote that follows is not from the passage text. That is, this is a New Information question.A: The new information in the question draws a distinction between literature on one hand, and sociology, politics, and

philosophy on the other. While Choice A is not inconsistent with that distinction, it goes in the opposite direction by describing the study of the relationship between literature and social and political conditions.

B: Yes. The new quote in the question stem claims that literature is different from other fields of study (sociology, politics, and philosophy). This theme is in agreement with the authorʼs claim in the passage that “Literary scholarship has its own valid methods” (line 48) and that literary study “is a species of knowledge or of learning” (lines 3–4). That is, both the new quote and this answer choice have the same theme: literature (as well as the study of literature) is to some degree different.

C: This statement goes too far; it is not an assertion made by the author in the passage. The claim that literature (which is an artistic pursuit, according to the passage [lines 2–3]) is distinct and unique doesnʼt indicate that other pursuits are not artistic. Nothing in the passage or in the new information in the question indicates that only literature is artistic, nor is it suggested that being artistic is what distinguishes literature from all other fields.

D: The theme of the new information in the question is the distinction between literature and other fields or forms of knowledge. The theme of this answer choice is the overlap or similarities between literature and science. As in Choice A, this answer choice, while not in contradiction with the new information, goes in the opposite direction (similarity rather than difference).

Passage III (Questions 94–99)

94. AA: Yes. The author discusses the “disquieting” (line 31) implications of the application of the concepts of natural

selection in paragraph 4. According to James Trefil, “natural selection as observed on Earth…channels intellect to predators” (lines 31–33). Trefil also states that “Most bright animals are carnivores” (lines 33–34). Therefore, the smartest beings may also be aggressive (line 30).

B: This is the right answer to the wrong question. While the passage does make this point in paragraphs 5 and 6, this is the comforting aspect of natural selection (lines 54–57), not the disquieting one.

C: While the passage does suggest that aliens might be more intelligent or knowledgeable than we are (lines 14–17 and 64–67) and that natural selection may “channel intellect to predators” (lines 32–33), the passage does not go so far as to suggest that natural selection would itself lead extraterrestrials to be more intelligent than humans. Humans, according to the passage, also are or have been aggressive predators (lines 37–41). Finally, the passage does not indicate that the prospect of intellectually superior aliens is frightening or disturbing.

D: As in Choice B, this is the right answer to the wrong question. That is, this statement is supported by the passage (lines 54–61), but this is the comforting, not the disquieting aspect of natural selection.

95. BA: If extraterrestrials land unannounced, we “would have little choice but to pay heed” (lines 1–3). Speculation about the

nature of aliens would be most crucial if we had a choice of whether or not to communicate (line 3–6).B: Yes. The author states that if aliens communicated with us by radio, and therefore we could decide whether or not

to reply, our “decision could turn on speculation about what the other beings were like” (lines 3–6).C: While our speculation would include the issue of whether or not the aliens were likely to be hostile, the passage does not

suggest that we would be most likely to speculate when they were in fact hostile (the question, in that case, would already be answered). Speculation would be most important in a case where we could decide whether or not to communicate (lines 1–6).

D: This choice takes words from the passage out of context. The author suggests that if aliens achieved space flight, “it could demonstrate that nuclear knowledge can be acquired without setting in motion Armageddon” (lines 18–21). However, the passage does not connect this issue to the issue of the circumstances in which speculation about the nature of extraterrestrials would be most crucial.

Page 18: AAMC MCAT Test 8 Solnsdocshare01.docshare.tips/files/23478/234782350.pdf · 2016. 12. 13. · MCAT Practice Test 8 Solutions 4 this fact from nuclear physics, you can still choose

MCAT Practice Test 8 Solutions18

96. CA: James Trefil suggests that aliens might be “aggressive, territorial, and quick to reach for the sword” (lines 26–31). The

statement that alien civilizations might avoid self-destruction through abhorrent means would support, not counter, Trefilʼs views.

B: Science-fiction writers, according to the passage, for the most part tell us about the supposed speech and dress of aliens (lines 7–11). The claim that aliens canʼt shoot straight isnʼt enough to tell us that they are aggressive, but it certainly doesnʼt indicate that they are peaceful. Therefore, the claim that aliens may have used violent or oppressive means to control war does not counter the views of science-fiction writers, as they are described in the passage.

C: Yes. The researcher quoted in the second paragraph argues that any civilization that makes it into space must no longer be warlike and aggressive; otherwise, they would have destroyed themselves (lines 12–17). The statement that alien civilizations “might avoid self-destruction by means abhorrent” (lines 22–24) is intended to counter this view by suggesting that a civilization might control war through aggressive or oppressive means. Therefore, as opposed to the researcherʼs claim, aliens who achieve space flight might still pose a threat to us.

D: This choice takes words out of context of the passage. Military tacticians are mentioned in line 52, but not as a contrast to the idea that war can be controlled through oppressive means. If anything, given that the author states that they find pacifist ideas to be “curious” or strange, we would expect that they might agree that other civilizations may display oppressive and aggressive qualities.

97. BNote: This question is essentially the reverse of question 94.A: The idea that extraterrestrials who contacted Earth may be the fittest of their species is not presented as a comforting idea.

The author suggests that natural selection may favor those who are “aggressive, territorial, and quick to reach for the sword” (lines 26–31).

B: Yes. This is essentially stated in the passage. The author writes that in some ways “the thought that natural selection might function on other worlds as it has on ours is comforting, for [it] would imply that ʻhuman ̓ nature was something deeper even than we know. Aliens…might exhibit may recognizable traits…” (lines 54–58).

C: According to science-fiction writers (in the authorʼs rather sarcastic description), aliens speak like Midwesterners, dress exotically, and have bad aim (lines 7–11). Nothing in this description is particularly disturbing; therefore, the comforting aspect of natural selection is not intended to counter the views promoted by science-fiction writers.

D: While the discovery of an alien civilization that used nuclear knowledge to achieve space flight without destroying itself in a nuclear Armageddon “might be cause for celebration” (lines 18–21), the implication is that this is because their example would suggest that we Earthlings may also be able to avoid blowing ourselves up. The author does not indicate that the aliens would explain to us how they achieved this feat. Finally, there is no connection in the passage between avoiding nuclear destruction and the comforting aspects of natural selection; they may even have avoided nuclear war “by means abhorrent” (lines 22–25).

98. AA: Yes. The possibility that they would be warlike is described in paragraphs 3 and 4. The possibility that they might

be peaceful is described in lines 12–17, and in paragraphs 5 and 6. The author does not take sides. Therefore, we could expect them to be either one or the other.

B: The author suggests that they could also be peaceful (lines 12–17 and 42–61).C: The author suggests that they could also be warlike (lines 21–41).D: If you arenʼt sure what “ethologically” means (“ethologically” in this context would mean “developmentally”), focus on

the word “backward”. The passage never suggests that aliens are likely to be in any way backward. The passage suggests instead that they may either be similar to humans (lines 54–61), or even ahead of us, if they have in fact “acquire[d] the wisdom to control war” (lines 12–17).

Page 19: AAMC MCAT Test 8 Solnsdocshare01.docshare.tips/files/23478/234782350.pdf · 2016. 12. 13. · MCAT Practice Test 8 Solutions 4 this fact from nuclear physics, you can still choose

19Verbal Reasoning Solutions

99. CA: James Trefil argues that “advanced extraterrestrials” may be predators who are “aggressive, territorial, and quick to the

sword” (lines 26–31). This is the opposite of pacifistic. B: The author describes both the view that aliens may be peaceful (lines 12–17 and 42–61) and the view that they could be

aggressive (lines 21–41). Given that the author does not take sides, we cannot infer that the author would favor a pacifistic characterization.

C: Yes. The researchers mentioned in the second paragraph have as “an article of faith” that “extraterrestrials will not be hostile” because they will have acquired “the wisdom to control war” (lines 12–17). The scenario in the question stem is entirely consistent with this view.

D: There is nothing in the scenario cited in the question stem about the aliens ̓manner of speech, their mode of dress, or their marksmanship. Therefore, it would neither support nor undermine the characterization favored by science fiction writers (lines 7–11).

Passage IV (Questions 100–104)

100. AA: Yes. The author states that under some conditions the size of the algae bloom “can increase rapidly” and “spread

over large areas” with catastrophic results (lines 28–32). Out of the four choices, this is the only one that is both supported by the passage and directly relevant to the issue of control of red tides.

B: This choice contradicts the passage. The author states that “the global expansion in aquaculture means that more areas are monitored closely” (lines 55–56) and that “the maturation of this field of science [is] now profiting from more investigators, better analytical techniques and chemical instrumentation, and more efficient communication among workers” (lines 60–63).

C: This choice does not focus on the same issue as the question stem. The effects of red tides on fish do not indicate why the tides themselves are difficult to control. Furthermore, the passage does not indicate that toxins are more likely to reach dangerous levels in small fish than in medium or large fish, or that small fish are in any relevant way different than larger fish.

D: The passage suggests that the opposite is true. See the explanation for Choice B, and lines 55–56 and 60–63. Furthermore, the author states that “Long-term studies…do show that red tides…are increasing as coastal pollution worsens,” in part due to “industrial, agricultural, and domestic waste” (lines 64–68). The existence of these long-term studies is further indication that human pollution is in fact being monitored.

101. DA: Given that algae blooms can spread rapidly over large areas (lines 26–32), this proposal would be unlikely to significantly

reduce the problem. It does not address the cause of red tides, and would at most offer limited protection to some commercially farmed fish (“fisheries ̓products” [line 57]).

B: Aside from the fact that there is no suggestion in the passage or question stem that we are able to control the movement of wild marine life, this would at most be a limited solution. As in Choice A, it would at most offer limited protection to some fish and marine mammals, without addressing the cause of the problem.

C: Dumping toxic substances (herbicides) into the ocean in order to kill other toxic substances (dangerous algae) would be a radical solution with possible unintended negative consequences. Also keep in mind that only some algae are toxic; herbicides would kill the harmless species as well. While this choice, unlike choices A and B, would in fact address the cause of the problem, it does so in an extreme and indiscriminate fashion. There is no evidence in the passage or question stem which suggests that environmentalists would advise such extreme measures.

D: Yes. The passage states that when wastes, which are “frequently rich in plant nutrients” (lines 67–68), are released into coastal waters, we see “a general increase in algal growth,” including toxic algae (lines 67–72). If plant nutrients were removed from wastewater, it would help address the cause of red tides without killing off all algae or introducing new toxins into the oceans.

Page 20: AAMC MCAT Test 8 Solnsdocshare01.docshare.tips/files/23478/234782350.pdf · 2016. 12. 13. · MCAT Practice Test 8 Solutions 4 this fact from nuclear physics, you can still choose

MCAT Practice Test 8 Solutions20

102. BA: According to the passage, “commercial fish are sensitive to [algal] toxins and, unlike shellfish, die before toxins reach

dangerous levels in their flesh” (lines 36–38). Therefore people who, unlike fish, birds, and whales, rarely eat the whole fish including the organs, are not at great risk of poisoning by red tides. Even if red tides increase, then, it does not appear that humans would be at significantly greater risk.

B: Yes. The passage states that “Algal toxins…cause mortalities as they move through the marine food web” (lines 33–34) because toxin “accumulates in the liver and other organs of certain fish, and so animals such as other fish, marine mammals, and birds that consume whole fish…are at risk” (lines 38–41). Therefore, fish may be killed through a direct encounter with a red tide (the author implies in lines 35–38 that this is true of “herring, cod, salmon, and other commercial fish”), or by eating other fish who have been exposed. Thus, if red tides increase as the author expects (lines 66–72), we would expect to see more large-scale fish die-offs.

C: The author states that “red tides…are increasing as coastal pollution worsens” (lines 64–65) and that pollution through nutrient-rich wastewater causes “a general increase in algal growth” (lines 68–70). Therefore, we would expect both toxic and harmless species of algae to increase (lines 70–71).

D: The author suggests that shellfish are less sensitive than are fish to algal toxins (lines 34–38). Therefore, we are more likely to see die-offs of fish (Choice B) than of shellfish. Furthermore, even if there were one or more large-scale die-offs, this would not necessarily mean that the total number of creatures would decrease. This choice is too extreme, especially when compared to Choice B.

103. CA: The question stem describes an actual epidemic of poisoning, not just groundless fears. If people were demanding more

medical attention to assuage their fear of being poisoned, it would not explain why people were poisoned in the first place. This choice has a problem with cause and effect. An epidemic could cause people to demand medical attention, but a demand for medical attention could not cause an epidemic.

B: The epidemic is described as worldwide. Even in countries with a seacoast, only a minority of people would be swimming in coastal areas. The most reasonable explanation has to explain how people in non-coastal areas are getting sick. Furthermore, the author states that “Algal toxins…cause mortalities as they move through the marine food web” (lines 33–34). Unless swimmers are eating significant amounts of algae, it isnʼt clear based on the passage that simple exposure would cause poisoning of humans.

C: Yes. The author states in the passage that people are generally not at high risk of red-tide poisoning because commercial fish are sensitive to toxic algae. Therefore, these fish tend to die before toxins accumulate in the parts of their bodies normally consumed by people (lines 34–38). If food fish became less sensitive (and so were more like shellfish [lines 37–38]), toxins would have more time to accumulate and reach dangerous levels in their flesh. This flesh could then be eaten by people, passing along the toxins.

D: There is no reasonable connection, based on the passage, between lower exposure of whales and other marine mammals to toxins on one hand, and an epidemic of poisoning among humans on the other.

104. CNote: The credited response will be the answer choice that is most inconsistent with the authorʼs concern about the danger of red tides. That is, we need a choice that goes the furthest to indicate that red tides do not pose a significant danger.A: This choice is too limited. While it may indicate that temperature fluctuations, not red tide, caused the death of fourteen

whales in 1987 (lines 1–4 and 12–13), this is not enough to suggest that red tides are generally harmless.B: As in Choice A, this finding would suggest only that there was an alternate cause (other than red tide) in one incident—the

death of fourteen whales in 1987 (lines 1–4 and 12–13). Invalidating one example does not significantly weaken the authorʼs overall argument.

C: Yes. This finding goes the furthest to suggest that red tides are not generally dangerous. In particular, it undermines the implication in lines 26–32 that red tides are likely to spread quickly over a large area, with disastrous results. If red tides are rare, and if when they occur they grow slowly and can be easily identified, then they do not appear to pose much of a threat.

D: According to the passage, it is “fish, marine mammals, and birds,” not humans, who are most at risk from toxic red tides (lines 34–41). Unless the antidote described in this answer choice can be easily administered to wild animals, the existence of an antidote would not suggest that the authorʼs concern is exaggerated. Furthermore, to the extent that humans would be at risk, the fact that they can be treated for poisoning does not suggest that poisoning should not be a concern.

Page 21: AAMC MCAT Test 8 Solnsdocshare01.docshare.tips/files/23478/234782350.pdf · 2016. 12. 13. · MCAT Practice Test 8 Solutions 4 this fact from nuclear physics, you can still choose

21Verbal Reasoning Solutions

Passage V (Questions 105–114)

105. CA: This choice is too narrow to express the primary purpose. The author does discuss two different critical accounts (lines

1–6). However, the author goes on to describe how these two accounts came together to create “a single conclusion” (line 7) or myth, and then spends the rest of the passage refuting that myth.

B: As in Choice A, this statement is too narrow. The author describes the growth of the magazine industry in paragraph 3 in order to support the larger point that pre-Civil War America was not hostile to the novel.

C: Yes. The author describes this idea (that America was hostile to both fiction in general and the novel in particular) in the first paragraph, lists three reasons why this idea is questionable in the second paragraph, and provides further support for the third of these reasons in paragraphs 3 and 4. The final paragraph sums up the main idea when it reiterates the point that the American public was not hostile to fiction in general or to the novel in particular (lines 66–70).

D: As with Choice A and Choice B, this choice is too narrow to be the primary purpose. The author does describe the argument that the Puritan tradition was one aspect (the nature of American social life is another) that fostered the romance and discouraged the novel in pre-Civil war America (lines 2–10) and then goes on to refute this argument (lines 16–21). However, this is just one part of the authorʼs overall purpose, which is to argue that pre-nineteenth century America was hospitable to fiction and to a particular form of fiction, the novel.

106. AA: Yes. The author discusses periodicals ̓reviews of novels (paragraph 4) in order to support the claim that “a great

many novels were written and published in America at this time of supposed hostility to fiction, and a great many more were being read” (lines 23–26). If it were easy and more profitable for American publishers to acquire European novels, and yet half of the reviews were still of American novels (lines 45–46), it would provide even more evidence that there was significant interest at that time in American novels (lines 48–53). That is, it would make the statistics cited in the passage even more compelling.

B: The author never discusses “literary nationalism.” Nor does the author indicate that a desire on the part of the American public to read American novels constitutes a form of nationalism. Even if it did (which would be reading too much into the passage), this answer choice indicates that the author argues that literary nationalism was not viable, which would contradict that interpretation.

C: If American publishers could make more profit by publishing European novels, and if they also made profit a top priority, one would expect that most published novels would be European in origin. However, the passage states that about half of the novels reviewed were American in origin. Therefore, the new information in the question indicates that profit was not a top priority.

D: See the explanation for Choice A. This information would support the authorʼs claim that the American public was interested in reading American novels, by suggesting that something other than profit motive must have been driving publishers ̓decisions to publish a significant number of novels written by American authors.

107. ANote: The correct answer either will not be an assertion made in the passage, or will be an assertion made in the passage but with no support by cited historical research.A: Yes. This choice is correct for two reasons. First, while the author does state in lines 64–66 that “the small number of

American fiction writers who are now called major did, evidently, have trouble supporting themselves as novelists,” the author provides no historical research to back up this claim. Note that the next word after this quoted sentence is “but”, indicating that the author is moving on to a different point, rather than providing further support or explanation. Second, the passage states that these writers had trouble supporting themselves as novelists; it does not, however, state that they had trouble supporting themselves by writing (that is, there could be ways of making money through writing other than by writing novels). For this reason, this answer choice is not even an assertion made by the passage author.

B: This assertion (made in lines 23–29 in the passage) is supported by the research about periodical reviews described in paragraphs 3 and 4.

C: This assertion is made and supported in lines 48–63.D: This assertion is supported by the research described in lines 29–41.

Page 22: AAMC MCAT Test 8 Solnsdocshare01.docshare.tips/files/23478/234782350.pdf · 2016. 12. 13. · MCAT Practice Test 8 Solutions 4 this fact from nuclear physics, you can still choose

MCAT Practice Test 8 Solutions22

108. BA: This sentence marks the end of the passageʼs discussion of Scottish common sense philosophy. Rather than introducing a

point that is discussed or developed further later on, it follows up on a point first raised earlier in the passage (lines 2–4).B: Yes. In lines 17–19 the author claims that “much of what Americans wrote and said about novels was derived from

sources written in the British Isles.” The sentence cited in the question stem further supports the claim that what Americans said about fiction to some degree derived from British sources (Scotland is part of Britain). Use Process of Elimination actively; while one could debate whether the role of lines 20–21 is to offer further support for the preceding sentence or to act as a conclusion based on the preceding sentence, there is no other answer choice that provides a more accurate description.

C: This statement is at odds with the main idea of the passage; that is, that nineteenth-century America was not hostile to fiction, including the novel. Note that the author refers to “supposed hostility to fiction” (line 25) and “expressed hostility to fiction” (line 16) [emphasis added]. The purpose of the sentence cited in the question stem is to further the argument that America was not uniquely or unusually hostile to fiction and the novel.

D: While the author may (or may not) believe that Scottish common sense philosophy was to some degree hostile to fiction, the sentence quoted in the question stem does not express that view. Rather, this sentence indicates that to the extent that any ideas drawn from Scottish philosophy appeared in or influenced American writings about the novel, those ideas cannot be described as essentially “American”.

109. BA: The new information in the question stem describes the idea that social conditions (breakdown of cohesiveness and increase

in individual isolation) are inconsistent with the conventional novel. There is no indication that “the Puritan tradition” relates to or is defined by social conditions. Therefore, Choice B, which does refer indirectly (“the nineteenth-century milieu”) to similar social conditions described in the passage, is a better fit.

B: Yes. The scenario described in the question stem parallels the myth described in the first paragraph of the passage. The author in the passage describes (and later refutes) the story that “the sparseness of American social life made the conventional novel difficult, even impossible to write” (lines 4–6) and that the “American imaginative space,” defined by “an ambience of isolation, alienation, defiance, and apology” lent itself to the literary form of the romance rather than to the novel (lines 7–12). That is, both the question stem and the passage describe the belief that when social conditions tend towards alienation and isolation, writers will be drawn away from the novel and towards some other literary form (such as the romance).

C: The “death-of-the-novel” theory is inconsistent with the research finding on book reviews described in paragraphs 3 and 4. Those findings, according to the author, indicate that Americans were interested both in reading and writing novels (lines 23–29, 49–53, and 68–70).

D: The author does not present research findings on attitudes towards the novel in England. While the author does suggest that there was some hostility to fiction in Britain (lines 16–17), this claim is not supported by research cited in the passage. Furthermore, there is no suggestion in the passage that British hostility was caused by social conditions or, more specifically, by an “ambiance of isolation” (line 11). Therefore, the new information is more consistent with Choice B, which does refer to social context (“the nineteenth-century American milieu”).

110. BNote: The correct answer will be the one that is most inconsistent with the passageʼs description in paragraphs 3 and 4.A: This is not inconsistent with the passage. The author states that there were “more than two thousand reviews of eight

hundred separate novels, about half of them American in origin” (lines 44–46) between 1840 and 1860. While the author argues that people in general had positive feelings towards the American novel, some did complain about overly positive reviews or “puffery” (lines 50–53); it would be consistent if some also complained about the number of reviews.

B: Yes. According to the authorʼs interpretation of the research cited in paragraph 4, “American novels were received warmly by reviewers” (lines 49–50); this warmness reflected the attitudes of Americans more generally, since “novel reviewing was conducted in constant awareness of what people were reading, and was directed toward trying to understand the reasons for public preferences” (lines 58–61). This is part of the authorʼs overall argument that nineteenth-century America was not hostile to fiction or to the novel (lines 23–29). If the information in the passage is correct, while an individual editorialist of the time could have been hostile to American fiction, we would not expect an editorial decrying American hostility to American fiction. This would be denouncing something that, according to the passage, did not exist.

Page 23: AAMC MCAT Test 8 Solnsdocshare01.docshare.tips/files/23478/234782350.pdf · 2016. 12. 13. · MCAT Practice Test 8 Solutions 4 this fact from nuclear physics, you can still choose

23Verbal Reasoning Solutions

C: This would not be unexpected. The author never suggests that all of the reviews of American novels originated in America. In fact, the author states in paragraph 2 that “much of what Americans wrote and said about novels was derived from sources written in the British Isles” (lines 17–19). Finally, the passage does not indicate that the British were in general hostile to American fiction.

D: This would not be unexpected. The author accepts that Scottish common sense philosophy influenced American writing to some degree (lines 17–21). There is no reason, based on the passage, to think that an essay reflecting this influence would not appear in an American periodical of this time period.

111. AA: Yes. Those who “set themselves up as guardians of critical integrity” complained that reviewers were overly positive,

i.e., writing “indiscriminate puffery” (lines 50–53). The author goes on to state that “novel reviewing was conducted in constant awareness of what people were reading” and that “reviewers had a fairly precise idea of what they were guiding and correcting” through their reviews (lines 58–63). These statements are intended to refute the potential objection that these positive reviews reflected only the opinions of the reviewers themselves (lines 53–57). Therefore, the complaints of the “guardians” provide credible evidence that the prevailing attitude of the public towards the American novel was quite positive.

B: The word “questionable” disqualifies this choice. The author argues that reviewers ̓positive views (“puffery” according to the self-styled guardians) reflected the views of the public as a whole. The author does not suggest, however, whether or not the positive reviews reflected the true quality of the books under consideration. For the guardians ̓ideas to be labeled as “questionable,” we would need some indication by the author that the book reviews may not in fact have been overly positive.

C: This choice goes even further in the wrong direction than Choice B. While there is no indication that the author finds the ideas of the self-appointed guardians to be “amusing,” the term “wrongheaded” is even more problematic. The author never suggests that the largely positive reviews did in fact reflect the true quality of the books reviewed, only that they reflected public opinion of the time. Thus there is no evidence that the author believes the guardians ̓claim that the positive reviews were “indiscriminate puffery” (overly positive) was incorrect.

D: Choices B, C, and D are all incorrect because they indicate that the author disagrees to some extent with the “guardians of critical integrity”; this choice (especially given the word “dangerously”) is the most extreme (and so the most clearly wrong) of the three. The author does not indicate that the novels that were positively reviewed were in fact good novels, only that the public as a whole believed that they were (lines 49–50 and 57–63). Therefore, there is no indication that the author believes that the “guardians of critical integrity” were incorrect in their belief that reviews were overly positive (“indiscriminate puffery” [line 52]). Thus, the passage does not suggest that the author believes the guardians ̓idea to be misinformed.

112. DA: This choice contradicts the passage. The author states in lines 21–23 that “the ʻconventional ̓novel at that time was only in

the process of becoming conventional.”B: The passage offers no information about how common the conventional novel was is England as compared to America.

The only direct comparison we have between the two countries is that “expressed hostility to fiction was no less strong in England than in America” (lines 16–17), but this tells us nothing about how common the novel was.

C: This choice contradicts the passage. In the first paragraph the author describes the argument that pre-Civil War America was inhospitable to the novel, and that the “American imaginative space” lent itself more readily to a different literary form, the romance (lines 9–12). Even though the author disagrees with the claim that America was hostile to the novel, the author does appear to agree that the novel and the romance are two different literary forms. Therefore, we would expect them to be different in style and/or subject matter.

D: Yes. The author states in the first paragraph that some literary historians believe that “the sparseness of American social life made conventional novels difficult, even impossible, to write” (lines 4–6). This suggests that novels are based on observations of social life, and that when there is little to observe, it is difficult to write novels. The author does go on to argue that novels were in fact quite common in this time period, but the passage never directly denies a connection between social life and the novel. Use Process of Elimination; this is not a great answer, but it is the best of the four choices.

Page 24: AAMC MCAT Test 8 Solnsdocshare01.docshare.tips/files/23478/234782350.pdf · 2016. 12. 13. · MCAT Practice Test 8 Solutions 4 this fact from nuclear physics, you can still choose

MCAT Practice Test 8 Solutions24

113. DItem I: True. This piece of information would be valuable in evaluating the accuracy of the authorʼs claim that the

generally positive reaction of reviewers to American novels represented a similar opinion on the part of the American public (lines 48–63).

Item II: False. While the author mentions that the reviews varied in length (lines 46–48), the author doesnʼt make an argument correlating or relating length of a review to popularity of the book. Therefore, this piece of information would not be relevant to evaluating the authorʼs conclusions.

Item III: True. The author refutes the claim made by literary historians that “The would-be American novelist before the Civil War was drawn, or forced, toward a literary form better suited to the American imaginative space: the romance” (lines 7–10). The author refutes this claim in part by citing evidence that many popular American novels were written in this time (lines 23–26 and 42–63). It would be useful to know, then, the popularity of romances among American readers; if romances were much more popular than novels, it would support the literary historians ̓story over that of the author. If, instead, novels were much more popular than romances, it would strengthen the authorʼs conclusions in the passage.

114. CA: The author never argues that the romance was not uniquely American. The only direct mention of the romance appears in

lines 10–12. While the author does go on to argue that the idea that the romance was better suited than the novel to America in that time period is a “critical myth” (lines 13–16), the author makes this argument by claiming that many novels were written in America, not by claiming or implying that many romances were written in other countries.

B: There is nothing in the new information or in the passage to suggest that Wuthering Heights is not a romance. Furthermore, there is nothing in the new information or in the passage to suggest that the literary historians mentioned in the first paragraph believe that romances were only written in America. Therefore, this new fact would not demonstrate that literary scholars are mistaken even occasionally, much less “often”. Finally, even if we did have evidence that literary historians are often mistaken, it would not significantly strengthen the passage authorʼs argument that literary historians are mistaken about the novel in nineteenth-century America (lines 1–16).

C: Yes. See the explanation for Choice A. The author makes no claims about whether or not romances were written in other countries. The only claim the author makes relative to the romance is that the romance was not better suited than the novel to the “American imaginative space” between 1840 and 1860 (lines 13–16).

D: Even if an atmosphere of alienation had spread to England from America, it would not undermine the passage argument. The author states that some ideas, for example Scottish philosophy, spread from England to America (lines 16–21); it would not be inconsistent if ideas flowed in the other direction as well. Furthermore, even though some literary historians connect the romance to an “ambiance of…alienation” (lines 10–11), one romance written in England would not constitute evidence that an atmosphere of alienation existed in that country.

Passage VI (Questions 115–122)

115. DA: This statement is consistent with the authorʼs claim that “by the late 1960s and early 1970s, limits on the energy base [in the

United States] began to surface” (lines 2–4). The author also mentions the decline in U.S. oil production after 1970 (lines 24–26).

B: This statement is consistent with the passage. The author states that “Environmentalism made its influence felt in a large number of ways” (lines 4–5) and then describes a variety of environmental legislation in lines 5–11. The passage also describes how environmentalism led to a switch from coal to low-sulfur oil (lines 12–16).

C: Although the author states that the fact that the U.S. “was outrunning its geological base…was not represented in either the consumption patterns or in prices” (lines 22–24), the authorʼs description of 1970 as a “turning point” when U.S. oil production peaked and then declined (lines 24–26) would be consistent with the statement that use of domestic oil has declined in recent years. This answer choice would also be consistent with the authorʼs claims that “an increasing number of individuals and communities in the U.S. are shifting to the soft path” (lines 40–41) and that low-sulfur oil is being imported (line 16).

D: Yes. This choice is inconsistent with the passage. The author claims that “A more rapid spread of [the soft path] approach is being hindered by government…subsidies of the hard-path approach” (lines 41–43).

Page 25: AAMC MCAT Test 8 Solnsdocshare01.docshare.tips/files/23478/234782350.pdf · 2016. 12. 13. · MCAT Practice Test 8 Solutions 4 this fact from nuclear physics, you can still choose

25Verbal Reasoning Solutions

116. DA: Lines 54–66 describe how large corporations are buying up copper mines and solar technology companies, while at the

same time disparaging solar energy. This paragraph has a negative tone, suggesting that the oil companies and other large corporations want to control the spread of solar technology in order to protect their own profits (lines 60–63). Therefore, the passage suggests that the author would oppose, not support, acquisition of solar technology companies by large corporations.

B: The passage suggests that the author supports increased use of soft energy technologies which “use to the greatest possible extent nondepletable sources like sun, wind, and vegetation…[and] emphasize diversification and dispersal of energy sources so as to avoid in the future the sort of dependence we now have on fossil fuels (lines 35–39). In particular, the author mentions solar energy as a promising source of energy (lines 48–52). Even though low-sulfur oil pollutes less than coal (lines 13–16), it is still a fossil fuel and part of the hard energy path.

C: See the explanation for Choice B. The author appears to support increased use of soft energy (lines 35–39 and 48–52). Nuclear reactors are listed as part of the hard path (lines 30–34).

D: Yes. The author criticizes the government for hindering the spread of the soft path by subsidizing the hard path (lines 41–43). The author also suggests that solar energy has the potential to supply a significant percentage of our energy needs (lines 48–52), and yet there is “inadequate access to capital for development of solar energy resources” (lines 46–47). Therefore, we can infer that the author would support federal subsidies for solar energy.

117. CA: This choice is inconsistent with the fact that “Concern about air pollution led to fuel switching, especially by electric

utilities, away from domestically produced coal to low-sulfur oil” (lines 13–16).B: The author never cites the environmental movement as insufficiently responsive; in fact, the environmental movement

appears to have had a significant impact on legislation (lines 4–11). Rather, the problem with shifting to a more environmentally friendly soft path lies with the government, building codes, lack of capital, false beliefs about solar energy, and large corporations (lines 41–52 and 58–66).

C: Yes. In the last paragraph, the author describes how oil companies are buying up copper mines, and how large corporations “began buying up solar technology companies” (lines 65–66) while they still “disparaged solar energy” (lines 59–60). By claiming that these actions were motivated by a concern that “every rooftop could become its own power plant and sensing that the cry for solar energy was a revolt against huge companies, utilities, and staggering electric bills” (lines 60–63), the author suggests that large corporations acted primarily to protect and maximize their own profits. Use Process of Elimination aggressively; this choice is a bit extreme, but it is the best supported of the four choices.

D: The author directly states that “a more rapid spread of [the soft path] approach is being hindered by government (taxpayer) subsidies of the hard-path approach” (lines 41–43). Therefore, even though the government has passed some pro-environment legislation (lines 5–11), there is evidence that the Congress is not fully committed to reform.

118. BItem I: True. Solar energy is part of the soft path (lines 35–36), and spread of this soft path is held back in part

by lack of capital (lines 46–47) and by government subsidization of hard-path technologies (lines 41–43). Therefore, we can infer that an advocate of the soft path would support tax credits for implementing solar technology.

Item II: True. An advocate of the soft path would certainly support legislation mandating a proportional increase in the use of soft energy.

Item III: False. Gas is listed as part of the hard-path solution (lines 30–33). We have no reason to think that soft-energy advocates would support switching from one hard-path resource to another. Be careful not to use outside knowledge or personal opinion. Also be careful to read the choice carefully though the last word; this choice sounds promising until the word “gas” at the very end.

Page 26: AAMC MCAT Test 8 Solnsdocshare01.docshare.tips/files/23478/234782350.pdf · 2016. 12. 13. · MCAT Practice Test 8 Solutions 4 this fact from nuclear physics, you can still choose

MCAT Practice Test 8 Solutions26

119. BA: This choice contradicts the passage. The author states, “In 1984, for example, about 18% of all primary energy used in the

world and 8.5% of that used in the U.S. came from renewable solar energy resources…” (lines 50–52).B: Yes. The author states that “Concern about air pollution led to fuel switching…away from domestically produced

coal to low-sulfur oil” and that this oil “had to be imported” (lines 13–16). While this is not a perfect answer (it assumes that all or most imported oil is low-sulfur and that non-domestic coal pollutes as much as domestic coal), it is the best of the four choices.

C: This choice is too extreme (and is more extreme than Choice B). The passage states that “oil companies now control almost 60% of domestic copper production in the U.S.” (lines 56–58). However, we know nothing about their control on a global level.

D: The author argues that the “usual proposed hard-path solution is the rapid expansion of three sectors” (lines 30–31), and gas is listed along with oil as the second of those three (lines 32–33). Even though more and more “individuals and communities in the U.S. are shifting to the soft path” (lines 40–41), many barriers are also described (lines 41–50). Therefore, we cannot infer that consumption of natural gas has declined.

120. DNote: If the soft-energy path is fully implemented, that means that hard-path energy is no longer used. Therefore, the correct answer will describe something connected to outdated hard-path energy use.A: The author lists wind as a soft-path resource (lines 35–36).B: Conservation is consistent with the soft-path (lines 41–45).C: Corn, an example of a “nondepletable resource like sun, wind, and vegetation” (lines 35–37), would be soft-path.D: Yes. Antipollution devices on smokestacks would be much less likely to be necessary when dealing with soft energy,

which the passage implies is less polluting than hard energy sources like coal and oil.

121. AA: Yes. The new information in the question relates to the theme in the last paragraph in the passage. In that paragraph,

the author describes how oil and other large companies have bought up copper mines and solar technology companies in order to control the implementation of solar technology. If an inventor perfected a solar battery, then, it would be likely that utility companies (mentioned or referred to in the passage in lines 58–59 and 62) would seek to control that technology.

B: The author claims that government subsidization of the hard-path is hindering spread of the soft-path, including solar energy (lines 41–43). Therefore, it is unlikely that the government would suddenly switch sides to subsidize solar energy.

C: The passage suggests just the opposite: “energy and high technology companies disparaged solar energy” and “spent a share of their public relations budget playing down the solar ʻmessiahsʼ ” (lines 58–65).

D: While this choice does not contradict the passage, at the same time there is no support for it in the passage text. Compare this choice to Choice A. We have an analogous case in the passage (paragraph 5) that makes Choice A likely to occur. On the other hand, the passage never describes lobbying efforts on the part of hard-path (including nuclear energy) industries. We donʼt know, in particular, that hard-path industries lobbied the government to obtain subsidies or to maintain “outdated building codes” (lines 41–46).

122. AA: Yes. While the passage does not discuss a relationship between price and consumption, this is the only choice that

is consistent with the new information in the question. That is, it is the only choice that describes a disadvantage attached to a hard-path resource. It makes sense that if the price of oil, a hard-path resource, rose precipitously, people would be more likely to seek out soft-energy alternatives.

B: If nuclear plants (hard-path [line 34]) were made safer, this if anything would encourage the continued or increased use of that hard-path resource.

C: If we could use coal without polluting, this would encourage, not discourage, the continued or even increased use of that hard-path resource.

D: The ability to fulfill our needs without having to import oil would not encourage us to seek elsewhere, in the soft-path, for energy sources.

Page 27: AAMC MCAT Test 8 Solnsdocshare01.docshare.tips/files/23478/234782350.pdf · 2016. 12. 13. · MCAT Practice Test 8 Solutions 4 this fact from nuclear physics, you can still choose

27Verbal Reasoning Solutions

Passage VII (Questions 123–127)

123. AA: Yes. The passage describes sports as the “most powerful manifestation” of “the godward signs in contemporary life

(lines 54–55), and as “manifestations of concern, of will and intellect and passion” (lines 60–62). The passage as a whole, in its description of sport as a “civil religion” (lines 1–2) leads up to this claim. Therefore, it is reasonable to conclude based on the passage that one should recognize the profound importance of sports. While this would be an even better answer choice if it mentioned the religious aspect of sport, even as written it is the best of the four choices.

B: While the passage does discuss how the ancient Olympics did honor the gods (lines 2–4), the passage as a whole does not discuss the religious history of sports. Rather, it discusses how sport has served a “religious function” (line 49) that may have little or nothing to do with “God” or gods (lines 55–59), or with institutionalized religion.

C: While patriotic impulses are mentioned in the first paragraph, the passage as a whole describes the importance of the religious function of sports. This choice is too narrow to express the primary message of the passage.

D: Nowhere in the passage does the author discuss the importance of distinguishing between these two aspects. Rather, ceremony (lines 42–49) and excitement (lines 45–46) are both mentioned or suggested, along with many other qualities, as aspects that are interwoven within the “natural religion” (lines 33–34) of sports.

124. BA: Be careful not to use outside knowledge or personal opinion. The author explicitly states that by “godward” he does

not mean belief in any concept of “God” (lines 54–57). Rather, “godward” suggests a drive towards “manifestations of concern, of will and intellect and passion” (lines 59–62). In the previous paragraph, the author refers to “a pagan sense of godliness” (line 53) as part of the “religious function” of sports (line 49). Therefore, the word “godward”, as used in the passage, does not refer to a “divine presence”.

B: Yes. Throughout the passage the author makes it clear that he is not using the word “religious” in the usual way; that is, to refer to a particular faith or form of worship. Rather, sports constitute a “natural religion” that can manifest itself in a wide variety of ways, from “the asceticism and dedication of preparation” to “a sense of participation in the rhythms and tides of nature itself” (lines 33–41). Even “agnostic persons” (line 48) can partake of this experience. The author takes great pains to show how all kinds of people can experience this religious impulse in a great diversity of forms. The authorʼs use of the word “godward” continues this theme, as the author writes, “I donʼt mean that participation in sports, as athlete or fan, makes one a believer in ʻGod, ̓under whatever concept, image, experience, or drive to which one attaches the name” (lines 55–59).

C: This choice is inconsistent with the passage. The author explicitly states that “godward ̓does not refer to the worship of “God,” as would be the case in conventional religion. Popular spiritual movements are never discussed by the author.

D: The author argues throughout the passage that sport arises from and itself constitutes a religious “godward” impulse that is distinct from religious movements and institutions. Therefore, attending sporting events would not lead to a revival of religion; attendance at sporting events would itself constitute “religious” practice.

125. CA: While the author states that the ancient Olympics and sports ever since have incorporated both political and religious

aspects (lines 2–7), he does not suggest that the athletes themselves are either religiously devout or patriotic. In fact, later in the passage the author states that “The athlete may of course be pagan” (lines 32–33). Finally, the author does not suggest that he would respect athletes for being either devout or patriotic.

B: The author never mentions skill of the athletes in this context.C: Yes. The first paragraph describes how the Olympics, from the ancient to the modern games, have been religious

and political as well as athletic events (lines 2–5 and 13–16). The rest of the passage argues that a religious impulse (not necessarily defined as a faith in a divine being) is a fundamental and essential aspect of sports. Therefore, by stating that the ancient games were held “in honor of the gods and…in honor of the state—and that has been the classical position of sports ever since” (lines 3–5), the author is describing the historical function of athletics.

D: The word “domination” is not consistent with the authorʼs description. Sports has always (since the ancient Olympics) incorporated both religion and politics, but the author does not indicate that athletics has been overshadowed by religion or politics.

Page 28: AAMC MCAT Test 8 Solnsdocshare01.docshare.tips/files/23478/234782350.pdf · 2016. 12. 13. · MCAT Practice Test 8 Solutions 4 this fact from nuclear physics, you can still choose

MCAT Practice Test 8 Solutions28

126. CNote: The credited response will be inconsistent with the authorʼs description of the religious impulse (as he defines “religious”) inherent in sports; it is this impulse which defines the importance of sports for the author. A: This choice is not inconsistent with the passage. The author states, “Most men and women donʼt separate the sections of

their mind. They honor their country, go to [a place of worship], and also enjoy sports” (lines 17–19). A finding that people prefer religious ceremonies to sporting events would not weaken the authorʼs argument that sports has its own religious aspects.

B: See the explanation for Choice A. The authorʼs argument does not depend on the capacity of sports fans to recognize the religious function of sports. In fact, the author suggests that most people do not “separate the sections of their mind” (lines 17–18); that is, they donʼt necessarily see the role played by religion and politics in sporting events.

C: Yes. If only conventionally religious people experienced the spiritual dimension of sports, this would suggest that the “natural impulse that is radically religious” (line 30) and the “godward” drive of sports (lines 54–60) do not include and apply to all participants, including those who do not believe in a “God” (lines 32–33 and 55–59). In this case, the religious value of sports would be much more limited than the author asserts.

D: This claim has no direct relevance to the authorʼs assertions about the value of sports; those assertions do not rest on the claim or assumptions that fans have stronger feelings (religious or otherwise) than the players. The author discusses the feelings of both (see lines 30–41 and 55–59), without prioritizing one over the other.

127. AA: Yes. Immediately before that sentence, the author states that “sports are manifestations of concern, of will and

intellect and passion” (lines 60–62). If losing is a symbolic form of death, this would highlight the level of concern and passion, or emotional investment, entailed in sports.

B: This choice is out of scope. The passage does not suggest that losers should be given equal honors.C: As in choice B, this answer choice is out of scope of the passage. Painful and unacceptable are not necessarily the same

thing. In fact, the author suggests a certain nobility to loss in this context (lines 60–63).D: The word “exaggerated” suggests a critical attitude that is not expressed by the author. The passage does not indicate that

experiencing a loss as a kind of death is inappropriate. In fact, the author indicates that the intense “religious” experience provided by sports serves an important function (lines 49–53).

Passage VIII (Questions 128–132)

128. CA: Only type-type identity theory is described as reducing mental states to physiological states (“every type of mental

[experience], such as pain, is identical to a corresponding type of physical [event], such as the firing of C-fibers” (lines 33–36). While behaviorism is described as simple (lines 12–13), it is described as reducing mental phenomena to “behavioral dispositions or tendencies” (lines 2–5), not to physiological states.

B: First, type-type identity theory does provide a reason for the existence of pain, that is, “the firing of C-fibers” (lines 33–36). Second, behaviorism provides a reason as well. A mental state such as “pain in the big toe consists of a set of dispositions to favor the big toe in certain ways” (lines 5–6) and “the belief that it is going to rain consists of dispositions to act as if it was going to rain” (lines 10–11); therefore, according to behaviorism, mental states or beliefs are constituted by dispositions to act in certain ways.

C: Yes. The author argues that behaviorism should be rejected in part because it ignores how beliefs interact with each other and with “various desires and other mental states” to create particular behavioral dispositions (lines 59–64). The author suggests that type-type identity theory has the same failing. In this theory, “every type of mental [experience]…is identical to a corresponding type of physical [event]” (lines 33–35). That is, one mental experience is caused by one physiological event; there is no consideration of how beliefs and feelings might interact with each other to lead to particular outcomes. This is one reason why type-type identity theory implies that the same (internal) physical event would cause the same mental event in any kind of creature (lines 36–39); the different contexts in which animals and people exist would be irrelevant. Note that type-type identity theory is distinguished from functionalism (beginning in line 39). Paragraph 5, which does describe taking context into account, is discussing functionalism, not type-type identity theory.

D: This is true, according to the author, of behaviorism (lines 17–24). However, type-type identity theory holds that mental events are caused or determined by internal physiological events (for example, the mental event of pain is caused by the physical event of the firing of C-fibers inside the body [lines 33–36]).

Page 29: AAMC MCAT Test 8 Solnsdocshare01.docshare.tips/files/23478/234782350.pdf · 2016. 12. 13. · MCAT Practice Test 8 Solutions 4 this fact from nuclear physics, you can still choose

29Verbal Reasoning Solutions

129. BA: According to the passage, behaviorists believe that pain (or, by implication, relief of pain) consists of certain behavioral

dispositions and actions (lines 2–11). In the new information in the question stem, pain relief is caused by the release of a substance within the body; nothing in this new information relates to the behaviorist explanation of mental events.

B: Yes. According to type-type identity theory, each mental event, such as pain, corresponds to a particular internal physical event (lines 33–36). The author also states that this implies that the same physical event would cause the same mental event, regardless of whether the physical event was occurring within an animal or a human being (lines 36–39). Therefore, someone who argues that because a substance that relieves pain in humans was found in earthworms, earthworms must be able to feel pain relief (and so, pain) would likely be a type-type identity theorist; one type of physical event corresponds to one type of mental event.

C: Functionalists, according to the passage, look at “a whole system of interdependent functions” (lines 49–51). That is, to understand beliefs and other mental states (including pain), “you have to understand [the way] beliefs function in relation to desires, intentions, perception, emotion, and inference” (lines 55–58). Unless all earthworms have the exact same desires, intentions, etc. as all humans, a functionalist would argue that finding a physical substance that relieves pain in humans tells us little about whether or not that substance corresponds to the same mental state (pain relief) in earthworms.

D: See the explanation for Choice B. While the author (who appears to support functionalism) would likely critique the conclusion described in the question stem (lines 36–39), the author does not go so far as to suggest that type-type identity theorists do not deserve to call themselves scientists. The author never argues that this approach is non-scientific.

130. AA: Yes. The author begins a critique of behaviorism in the second paragraph. The passage describes the view of

“many critics” who believe that “behaviorism takes too external view of the mental” and that “mental states and processes are not behavioral dispositions; they are the internal states and processes that are responsible for relevant behavioral dispositions” (lines 17–22). It becomes clear in the following paragraph that the author agrees with these critics when the passage states that “Now it seems clear that states and processes [of] the brain are responsible for behavioral dispositions” (lines 23–24). The author continues this critique in the final paragraph, when he writes that “crude behaviorism must fail” because it does not consider how “relevant behavioral dispositions are determined by various beliefs taken together plus various desires and other mental states” (lines 59–64). That is, behaviorism fails to consider this internal context.

B: The author criticizes type-type identity theory by suggesting that it presents an “overly strong thesis because of its implication that the physical basis for any type of mental [experience] must be the same in different creatures” (lines 36–39). The author follows this with a description of “a [weak form of] functionalism”; it becomes clear in the final two paragraphs that the author tends to agree with functionalism. In particular, the author uses notions proposed by functionalism to criticize behaviorism (lines 64–69).

C: Predictable actions are never discussed in the passage.D: According to the author, “crude behaviorism” accepts that feelings and beliefs do exist, and claims that all mental states

including beliefs and feelings “are behavioral dispositions or tendencies” (lines 2–5). For example, a behaviorist would claim that “the belief that it is going to rain consists of dispositions to act as if it was going to rain” (lines 10–11).

Page 30: AAMC MCAT Test 8 Solnsdocshare01.docshare.tips/files/23478/234782350.pdf · 2016. 12. 13. · MCAT Practice Test 8 Solutions 4 this fact from nuclear physics, you can still choose

MCAT Practice Test 8 Solutions30

131. ANote: The credited response will be the choice that is most inconsistent with the passage.A: Yes. This choice describes a conclusion that would be drawn by a behaviorist, not by the author. Behaviorism

identifies beliefs with actions. According to this theory, if the climber accepts another rope (action), that would constitute evidence for a particular belief (weakness of the original). The author criticizes behaviorism in part by arguing that “A particular belief taken by itself cannot be identified with particular behavioral dispositions because the relevant behavioral dispositions are determined by various beliefs taken together plus various desires and other mental states” (lines 60–64). The example of the umbrella (lines 64–69) continues this argument that a single action may be determined by a whole set of interrelated beliefs and desires. Therefore, the author would be unlikely to agree with the conclusion described in this choice.

B: The author would likely agree that a particular belief (weakness of the original rope) may lead to a variety of different results, depending on how that belief interacts with other beliefs and desires (lines 60–69).

C: As in Choice B, this conclusion is consistent with the authorʼs argument that actions or behavioral dispositions (taking the new rope) are determined by the interaction of a variety of different beliefs and desires (lines 60–69). Therefore, the author would agree that a belief in the weakness of the original rope would not by itself determine the climberʼs decision.

D: The author would agree with this conclusion; the passage states that “it seems clear that states and processes [of] the brain are responsible for behavioral dispositions” (lines 23–24).

132. AA: Yes. If there is a need for a belief, that belief would perform a certain function, and “Functionalism holds that

mental events of a certain sort are to be identified as those underlying events, whatever they are, that function in the relevant way” (lines 42–44). While the author uses examples of beliefs we would tend to see as explicit to describe functionalism in the passage (e.g., “your…belief that umbrellas are good for keeping rain off” [lines 67–68]), nothing in the passage suggests that functionalism is limited to explicit beliefs or information. Note: In this context, “implicit beliefs” would refer to beliefs that are only articulated when a relevant occasion arises. People donʼt generally walk around with the well-formed belief that giraffes donʼt wear hats in the wild because most people would never have considered the issue. However, most would agree that wild giraffes donʼt wear hats, based on general knowledge, if a response to the question of whether or not giraffes wear hats were required.

B: This response is consistent with type-type identity theory (lines 33–39), not functionalism. According to functionalism, beliefs are neither dispositions to behave (behaviorism) nor underlying physical events (type-type), but instead are mental events that in conjunction with other mental events determine or cause behavior (lines 42–48 and 60–64).

C: First, the author does not distinguish between implicit and explicit. Second, the author argues that beliefs are quite important within functionalism (lines 62–64), along with desires, intentions, perception, and feelings (lines 55–58), that is, along with other mental states.

D: As described in the question stem, the implicit belief that wild giraffes donʼt wear hats does have a function; it allows someone to answer the question of whether or not giraffes wear hats in the wild. The author does not suggest in the passage that functionalism distinguishes between implicit and explicit beliefs, and beliefs in general play an important role in the functionalist view (lines 62–69).

Passage IX (Questions 133–137)

133. CA: This choice is inconsistent with the passage. “Underusers” are people who “rarely use their learned competence or perhaps

those whose learned competence is low” (lines 44–46). It would make little sense to say that learned rules are most useful when learned rules are not being used.

B: The passage directly states that learned rules or “such (cognitively based) knowledge is usually not very helpful in normal communicative situations, particularly in the beginning stages” (lines 32–34).

C: Yes. The author writes, “Most speakers meet conditions for monitoring regularly and systematically only on cognitive grammar tests” (lines 25–27). Monitoring is defined as editing oneself “to make minor changes or corrections” based on learned rules (lines 15–19).

D: This choice contradicts the passage. The author states that learned rules are “usually not very helpful in normal communicative situations” (lines 32–34), like conversations.

Page 31: AAMC MCAT Test 8 Solnsdocshare01.docshare.tips/files/23478/234782350.pdf · 2016. 12. 13. · MCAT Practice Test 8 Solutions 4 this fact from nuclear physics, you can still choose

31Verbal Reasoning Solutions

134. BA: “ ʻSuperusers ̓are “those who are consciously able to apply learned rules quickly and efficiently so that a listener would

not notice the monitoring at all” (lines 56–58). Someone who frequently corrected him or herself while speaking would not qualify as a superuser; the monitoring would be noticed.

B: Yes. “ ̒ Overusers ̓spend so much time and effort on correctness that it often seriously interferes with communication” (lines 48–50). Someone who frequently corrected his or her own grammar while speaking would qualify as an overuser.

C: “ ̒ Optimal users ̓are those who are able to monitor their speech and improve their level of grammatical accuracy, but not to such an extent that it interferes with smooth communication” (lines 51–54). It is likely that frequent self-correction would in fact interfere with smooth communication.

D: “Underusers” are “those who rarely use their learned competence or perhaps those whose learned competence is low” (lines 44–46). One would expect an underuser to rarely if ever correct his or her own grammar while speaking.

135. ANote: The credited response will be inconsistent with some claim made by the monitor theory, as that theory is described in the passage.A: Yes. The monitor theory as formulated by Krashen and others indicates that “students who wish to communicate

must acquire this ability in much the same way that speakers, adults or children, acquire it in natural situations” (lines 12–15). Furthermore, “learned, rather than acquired rules are of limited use to the student; for some, they serve as a monitor—i.e., primarily an ʻeditor ̓to make minor changes or corrections in utterances which for the most part are initiated by acquired knowledge” (lines 15–20). The passage also states that according to the monitor theory, “Most speakers meet conditions for monitoring regularly and systematically only on cognitive grammar tests” (lines 25–27) and so that “oral proficiency in communication is not necessarily related to the ability to achieve high scores on standard grammar tests” (lines 35–37). If the scores of beginning students on grammar tests did in fact predict their oral proficiency, this would cast doubt upon the monitor theoryʼs claim that learned rules are only useful for grammar tests. It would also indicate that grammar tests may be a good way of evaluating students ̓progress (lines 37–41). Finally, it would also suggest that learned rules are in fact useful in the beginning stages of learning a second language, as opposed to the monitor theoryʼs claim to the contrary (lines 32–34).

B: See the explanation for Choice A. This choice would strengthen, not weaken the claim that learned rules are usually only useful on grammar tests (lines 25–27).

C: “Superusers” are unusual in that they can seamlessly monitor their own speech when necessary (lines 55–58). “Many language instructors fall into this category” (lines 58–60). One would expect these instructors ̓scores on grammar tests to match their oral proficiency. The author states that “many, perhaps most, students are not capable of performing with the mental gymnastics of their supermonitor instructors” (lines 64–66).

D: While “many language instructors” fall into the “superuser” category, some or many others would not. For those who are not superusers, we would expect that their scores would be poor predictors of oral proficiency (lines 34–37). This finding is not inconsistent with the monitor theory.

136. AA: Yes. In the beginning of the passage, the author describes “acquisition” as “the unconscious formulation of

grammatical principles” (lines 3–4). The author goes on to suggest that “students who wish to communicate must acquire this ability in much the same way that speakers, adults or children, acquire it in natural situations” (lines 12–15). Therefore, underusers (those who rarely use learned grammatical rules [lines 44–46]) may still learn to speak well through acquisition, that is, to learn like children through natural experience.

B: Underusers rarely use learned rules of grammar (lines 44–46). The fact that learned rules are not particularly useful does not explain how underusers can in fact become competent speakers through some other route (acquisition). That is, the fact that underusers do not use a relatively useless means to an end does not explain how they eventually succeed in achieving that end.

C: People who can seamlessly apply rules to monitor and correct their own speech would be superusers (lines 56–58). This information does not help us to understand how underusers (on the opposite end of the spectrum) might achieve “high levels of communicative accuracy.”

D: Modifications to utterances, as described in the passage (lines 17–20) entail applying learned rules to acquired knowledge. Given that underusers rarely use learned rules (lines 44–46), this information does not explain how underusers may still be able to learn to communicate well.

Page 32: AAMC MCAT Test 8 Solnsdocshare01.docshare.tips/files/23478/234782350.pdf · 2016. 12. 13. · MCAT Practice Test 8 Solutions 4 this fact from nuclear physics, you can still choose

MCAT Practice Test 8 Solutions32

137. CA: The passage suggests that conscious learning is not an effective way of learning a second language (lines 7–17 and

28–34). The author specifically states that learning rules through “drill” (line 30) “is usually not very helpful in normal communicative situations, particularly in the beginning stages” (lines 32–34). Memorization of vocabulary through drilling would therefore be unlikely to improve fluency.

B: The author suggests that the best way to become fluent, including the acquisition of grammatical rules, is to “acquire [this ability] in natural situations” (lines 11–15). At the end of the passage, the author also states that most adults can acquire grammatical rules if they “interact in communicative situations with native speakers of the target language” (lines 68–72). There is no natural situation or interaction entailed in using an audiotape while sleeping. This choice may be tempting because “listening” while asleep may appear to engage the unconscious rather than the conscious mind, and the author indicates that the unconscious acquisition of grammar is much more effective than conscious learning (lines 3–10). However, the author never discusses this particular form of unconscious learning, and it is still missing the crucial element of using a language in a natural and interactive context.

C: Yes. The author states that “students who wish to communicate must acquire this ability in much the same way that speakers, adults and children, acquire it in natural situations” (lines 12–15). At the end of the passage the author comes back to the same theme, stating that most adults “acquire rules readily, although usually imperfectly, given the chance to interact in communicative situations with native speakers of the target language” (lines 68–72). While they may not achieve perfection, using language in an interactive, natural situation (in this answer choice, by asking questions) is, according to the author, the best way to attain fluency.

D: The author argues that memorization (conscious learning) is not an effective way of learning a second language. Acquisition, which is the “unconscious formulation of grammatical principles” is the more effective route (lines 1–10). This acquisition is most likely to occur in a natural situation (lines 12–15) when students can interact with native speakers (lines 70–72); this would not be the case when a student is performing mental exercises to improve memory.

Page 33: AAMC MCAT Test 8 Solnsdocshare01.docshare.tips/files/23478/234782350.pdf · 2016. 12. 13. · MCAT Practice Test 8 Solutions 4 this fact from nuclear physics, you can still choose

33Biological Sciences Solutions

BIOLOGICAL SCIENCES SOLUTIONS

Passage I (Questions 138–143)

138. D. The passage states that rickets is a disease causing inadequate mineralization of bone, and is due to insufficient vitamin D activity. It also states that parathyroid hormone leads to bone breakdown. Thus, Statement I is false: a deficiency of parathyroid hormone would not lead to inadequate mineralization, it would lead to less bone breakdown. This eliminates answer choices A, B, and C, leaving D as the correct answer. Statements II and III are true; both an inability to convert vitamin D to its active form and the inability of active vitamin D to affect its target tissue would lead to inadequate mineralization. Note that this is a common theme on the MCAT: a faulty messenger or a faulty receptor both lead to the same physical symptoms.

139. B. The passage states that vitamin D stimulates the absorption of calcium from the small intestine (choice B is correct and choice D is wrong). Since calcium supplements are typically swallowed, including vitamin D in the tablet would enhance the uptake of that supplement. Choice A is true but irrelevant; it supports the need for vitamin D supplementation, but does not explain why vitamin D should be included in a calcium supplement. Activated vitamin D enhances parathyroid hormone activity, not calcitonin activity (choice C is wrong).

140. B. Statement I is true; enhancing osteoclast activity will increase bone breakdown thereby releasing more calcium into the blood. Statement II is also true; parathyroid hormone is the trigger for enhanced osteoclast activity, and it is released when blood calcium levels are low. Statement III is false; although vitamin C is required for adequate bone formation, it has no role in regulating blood calcium levels.

141. C. Calcitoninʼs job is to decrease bone resorption. If the level of calcium in the plasma is high, there is no need to resorb bone; thus, calcitonin is released. A dietary deficiency of either calcium or vitamin D would lead to low plasma calcium levels, and under those conditions bone resorption would need to remain high (choices A and B are wrong). Because parathyroid hormone and calcitonin have opposite effects, calcitonin cannot compensate for low parathyroid hormone levels (choice D is wrong).

142. B. A child with rickets has a vitamin D deficiency. The passage states that vitamin D is nonpolar, so dietary deficiencies in fat-soluble vitamins would most likely result in vitamin D deficiency (choices C and D are wrong). Furthermore, a child living in a northern climate is more likely to develop rickets than a child living in a tropical climate, because the child in the tropical climate would have enough sunlight available to stimulate vitamin D production by the skin. This could compensate for a lack of vitamin D in the diet (choice B is correct, and choice A is wrong).

143. A. Removal of the parathyroid glands would result in a lack of parathyroid hormone, which is needed to increase plasma calcium levels when they are low. Calcium is required for the proper function of neurons (neurotransmitter release) and muscle (excitation–contraction coupling). If the parathyroid glands were removed, the body would not be able to fix low plasma calcium levels, and neurons and muscles would cease to function properly. Calcitonin has opposite effects in the body from parathyroid hormone and cannot compensate for its loss (choice B is wrong). Drastic changes in the mineral-to-matrix ratio describes rickets, which is not caused by a lack of parathyroid hormone (choice C is wrong). There would not be an accumulation of calcium in the plasma; if anything, there would be a deficiency (choice D is wrong).

Passage II (Questions 144–150)

144. B. The best evidence for the necessity of something in the body is obtained by taking away that thing and seeing if the effects are still present. Thus, repeating the experiment in an animal that lacks neutrophils and observing the amount of resulting tissue damage would best clarify the role that neutrophils play in this process. Repeating the experiment with B-cell depleted animals would give evidence as to the role of B-cells but would not give information regarding the role of neutrophils (choice A is wrong). Repeating the experiment with different antibodies is a variation of the original experiment and will give more evidence, but the evidence would not be as strong as in the experiment removing the neutrophils altogether (choices C and D are wrong).

Page 34: AAMC MCAT Test 8 Solnsdocshare01.docshare.tips/files/23478/234782350.pdf · 2016. 12. 13. · MCAT Practice Test 8 Solutions 4 this fact from nuclear physics, you can still choose

MCAT Practice Test 8 Solutions34

145. A. The passage states that the adherence of neutrophils was facilitated by a receptor on the neutrophil, and that adherence resulted in tissue damage. Antibodies against the β subunit prevented tissue damage, implying that it prevented adhesion, while antibodies against the α subunit had no effect (neutrophils must still have been able to adhere). There is no reason to assume that proteinases are transferred to the neutrophils; if anything, proteinases are being transferred from the neutrophils to the surrounding tissue, resulting in damage (choice B is wrong). Since the bonding between the two subunits was described as noncovalent, it is safe to assume it involves hydrogen bonding; however, this would not be the sole function of the β subunit (choice C is wrong). Since this is a normal receptor on the neutrophil surface, it should not result in antibody formation in a normal animal (antibody formation should occur only in the presence of foreign or abnormal proteins, choice D is wrong).

146. D. The graph shows that a reduction in tissue injury occurred only when the antibody was received before reperfusion (note that “before ischemia” implies before reperfusion). Animals that received the antibody during reperfusion were not helped; they still had a high percentage of tissue injury (choice D is correct, and choices A and C are wrong). Applying the antibody after reperfusion was not tested in this experiment (choice B is wrong).

147. B. Antibody B is directed against the receptor that leads to adhesion and release of toxic products. The antibody would bind to the receptor, preventing adhesion and the subsequent effects. The passage offers no information about the affinity of the antibody, and further, the affinity of an antibody has nothing to do with whether or not it would be rejected by the patient (choice A is wrong). It is true that antibodies are very specific, so an antibody directed against the β subunit will not bind anything other than the β subunit. However, this does not explain why it would be better than free-radical or protease inhibitors (choice C is wrong). The passage offers no information about the half-life of antibody B (choice D is wrong).

148. B. The human body will generate antibodies against any foreign protein, including mouse antibodies. However, initial responses to foreign proteins are generally weak compared to subsequent responses to the same foreign protein; thus, with repeated usage the human response to the mouse antibody would get more and more severe, even if the initial response was weak (choice B is correct, and choice A is wrong). Antibodies will recognize whatever they are generated against, regardless of the organism they were generated in (choice C is wrong). The passage demonstrates the use of the antibody in vivo (choice D is wrong).

149. B. Pus results when neutrophils destroy bacteria by phagocytosis. It is a mix of dead cells and cellular debris. Thus, the presence of pus indicates the presence of neutrophils doing their job, and in order for the neutrophils to be present, they must have functional adhesion receptors (choice B is correct, and choice A is wrong). The amount of circulating red blood cells has nothing to do with neutrophil adhesion receptors (choice C is wrong), and if the neutrophil adhesion receptors were defective, more white blood cells would be circulating because less would be adhering (choice D is wrong).

150. C. If the neutrophils can migrate through vascular endothelium, they must still be able to bind to the endothelium. If they are binding, they are probably still able to release their toxic products (choice A is wrong). Since we know that the β subunits are necessary for binding etc., we can assume the β subunits are functional, but this does not tell us that only the β subunits are functional (choice B is wrong). If the cells are migrating, they must have a functional cell membrane (choice D is wrong).

Passage III (Questions 151–155)

151. C. Kinetic control means the product that forms faster is the major product; the rate at which a product is formed is dependent on the activation energy for the reaction. Therefore, the product with the lower activation energy is formed faster. Choice D states exactly the opposite, and can therefore be eliminated. Choice B is a false statement since rate of formation is dependent on the activation energy required to form a specific transition state. Choice A can be eliminated since melting point has no connection to rate of formation of a product.

152. C. Thermodynamic control means that the more stable product will be formed. Choice D can be eliminated; if the molecule has a higher potential energy, it is less stable. Choice A is a trap since it indicates the more stable product, but the more stable product may or may not form faster. The kinetic product will always form faster. While choice B may also sound good because generally speaking, more substituted alkenes are more stable, the alkene is not the only part of the molecule that may contribute to stability.

Page 35: AAMC MCAT Test 8 Solnsdocshare01.docshare.tips/files/23478/234782350.pdf · 2016. 12. 13. · MCAT Practice Test 8 Solutions 4 this fact from nuclear physics, you can still choose

35Biological Sciences Solutions

153. A. Nuclear magnetic resonance splitting patterns are determined by the n + 1 rule, where n is the number of Hʼs on the adjacent carbon. Since the aldehyde proton has no neighboring Hʼs, we have 0 + 1 = 1, a singlet.

154. B. Washing with water will remove soluble impurities, not insoluble ones, so choice B (not C) is the answer. Choice A can be eliminated since the product has already been recrystallized, and choice D is wrong because spreading the crystals evenly over the surface of the funnel is irrelevant.

155. C. Carbonyl carbons have a double bond to the oxygen and two other single bonds, for a total of three electron groups around the atom. Therefore, three hybrid orbitals are needed, so sp2 is the best choice.

Passage IV (Questions 156–160)

156. A. Excessive aldosterone would act on the kidneys, directly counteracting their ability to increase fluid output. Aldosterone causes an increase in the reabsorption of sodium and indirectly increases the reabsorption of water. This would prevent water loss through the urine, increase blood volume, and keep blood pressure high. An excess of glucagon and a shortage of insulin would both lead to excess glucose in the bloodstream, which would act osmotically to increase blood volume and therefore blood pressure. However, the kidneys could still help by increasing urine output, as nothing is inhibiting that function under these conditions (choices B and D are wrong). A shortage of thyroxine would most likely not affect blood pressure at all, but if it did, it would probably lead to a decrease in blood pressure via a decrease in heart rate and general body metabolism (choice C is wrong).

157. D. Since filtration is directly proportional to blood pressure, an increase in blood pressure necessarily produces an increase in filtration rate and a subsequent increase in urine production. The bladder has nothing to do with urine production; it is simply a storage tank for urine, and increasing its blood flow would have no effect on urine output (choice A is wrong). Increased reabsorption of solutes and water would reduce the amount of water in the urine (reabsorption means to return a substance to the bloodstream), not increase it (choice B is wrong). Likewise, increasing the permeability of the collecting duct to water would result in increased water reabsorption and decreased urine output (note that this is the effect that ADH—antidiuretic hormone—has on the nephron, choice C is wrong).

158. C. Hypothesis B essentially says that increased blood pressure (the problem) leads to increase urine output and decreased blood volume (one possible solution). Thus, the inverse must also be true: a decrease in blood pressure should lead to decreased urine output and increased blood volume and pressure. The kidneys monitor pressure at the glomeruli because decreases in glomerular pressure can reduce the rate of blood filtration, and this is potentially deadly. The application of clamps to the renal arteries would result in a slight initial increase in blood pressure (due to the small increase in vascular resistance at these arteries), but the lack of blood flow to the glomeruli would drop glomerular pressure and severely restrict filtration. In an effort to overcome that problem, the kidneys would increase water reabsorption to boost blood volume and systemic blood pressure in an effort to increase filtration (this is the sustained rise in blood pressure seen after a while). Thus, the drop in glomerular blood pressure led to sustained hypertension. This is what is expected and therefore Hypothesis B is supported. Although the clamps would lead to an initial increase in blood pressure, they cannot account for the sustained gradual increase (choice A is wrong), and the fact that the kidneys are receiving less blood has nothing to do with Hypothesis A (choice B is wrong). There is no reason to assume that the volume of body fluids was decreasing (choice D is wrong).

159. A. Consider the equation in Hypothesis A: P = CO × VR, which can be rewritten as P/VR = CO. The question states that P doubled and VR increased by 50%, and wants to know what happened to CO. Since (2P)/(1.5VR) = (4P)/(3VR) = (4/3)CO, we find that CO increased by 1/3.

160. B. Hypothesis A discusses changes in vascular resistance, which is brought about by contraction and relaxation of vascular smooth muscle. Striated muscle is skeletal muscle, and is multinucleate (choices A and D are wrong). Cardiac muscle is found only in the heart (choice C is wrong).

Page 36: AAMC MCAT Test 8 Solnsdocshare01.docshare.tips/files/23478/234782350.pdf · 2016. 12. 13. · MCAT Practice Test 8 Solutions 4 this fact from nuclear physics, you can still choose

MCAT Practice Test 8 Solutions36

Free-Standing Questions (161 through 164)

161. C. Antibodies are secreted proteins and are therefore synthesized by ribosomes on the rough ER. The nucleus and mitochondria have nothing to do with protein synthesis (choices A and B are wrong), and the Golgi apparatus is associated only with packaging of secreted proteins, not their synthesis (choice D is wrong).

162. A. The “knee-jerk” reflex is an example of a simple stretch reflex and as such does not require the input of the brain (choices B and C are wrong). Further, only one motor neuron is required to stimulate a skeletal muscle, not a series of motor neurons (choice D is wrong).

163. D. In order to remove triethylamine from an organic solvent into the aqueous layer, you need to protonate the nitrogen and give the molecule a charge. Therefore, treat with an acid.

164. C. The predominant conformation of the molecule will have the tert-butyl group in an equatorial position (since it is so sterically large) in order to minimize 1,3-diaxial interactions. Eliminate choices A and B. Axial and equatorial positions alternate as you move around the ring for substituents on the same face. Therefore, the chlorine must be axial.

Passage V (Questions 165–169)

165. A. The only place in the nephron where filtration occurs is in the glomerulus. The loop of Henle establishes the medullary concentration gradient and helps to reabsorb water (choice B is wrong), the distal convoluted tubule helps to “fine tune” urine by reabsorbing necessary ions (for example, sodium—this is where aldosterone has its effect—choice C is wrong), and the proximal convoluted tubule is where most filtrate modification—reabsorption and secretion—occurs (choice D is wrong).

166. B. The passage states that colchicine is an inhibitor of microtubule reorganization, a process that is necessary for phagocytosis. Preventing phagocytosis would decrease inflammation and reduce pain. Diffusion through cell membranes is not affected by microtubules (choice A is wrong), nor is uric acid crystal formation (choice C is wrong), nor is pH maintenance (choice D is wrong).

167. C. The passage states that uric acid is formed by the breakdown of purines to xanthine. The only purine in the answer choices is guanine. Cytosine, uracil, and thymine are all pyrimidines (choice C is correct, and choices A, B, and D are wrong.).

168. A. The passage states that the patient had normal levels of PRPP synthetase, so transcription and translation must not be affected (choices C and D are wrong). Since the pH optimum and enzyme activity are normal when the enzyme is purified, the active site must not be affected (choice B is wrong). The only time this enzyme malfunctions is when it is in the patient, where its activity is stated to be three times normal levels. This indicates that there must be some other molecule in the patientʼs cells that is affecting the activity of this enzyme, and not by binding to the active site since product formation is high, but is otherwise unaffected. Molecules that affect enzyme activity without binding to the active site are called allosteric regulators (choice A is correct).

169. B. Flying birds must remain light; they have hollow bones and little body mass. Excess water would contribute to the weight of the bird, so it would be an advantage for flying birds to excrete nitrogenous waste in the form of uric acid, instead of ammonia or urea, which require water. Wild pigs typically inhabit areas where water is available (forests, as opposed to deserts) so they can (and do) eliminate nitrogenous wastes as urea (choice A is wrong). Sharks and fishes eliminate nitrogenous waste as ammonia, which is highly toxic and must be diluted in large amounts of water. However, since they live in water, this is not a problem (choices C and D are wrong).

Page 37: AAMC MCAT Test 8 Solnsdocshare01.docshare.tips/files/23478/234782350.pdf · 2016. 12. 13. · MCAT Practice Test 8 Solutions 4 this fact from nuclear physics, you can still choose

37Biological Sciences Solutions

Passage VI (Questions 170–174)

170. A. The strength of an acid is related to the stability of its conjugate base. Since the negative charge on the nitrogen that results from the proton abstraction can be stabilized by resonance with the adjacent carbonyl π bond, A is the best answer. Choice B can be eliminated since polarity does not necessarily correlate with acidity. Similarly, just because the amide is not basic does not mean it is acidic. Eliminate choice D. An electron-donating aromatic ring would make the amide hydrogen less likely to leave, and the functional group less acidic. However, because of the nitro group on the benzene ring, the substituent is electron withdrawing. In either case, C is therefore incorrect.

171. C. As shown in Scheme 1 of the passage, the carbonyl of the amide group is lost in the Hofmann rearrangement, and a new bond is formed between the alpha carbon and the nitrogen. Only choice C shows loss of the entire carbonyl while keeping the nitrogen present in the molecule.

172. D. Hydrolysis of a carboxylic acid derivative yields the carboxylic acid itself. Choice A shows a reaction in which the nitro group on the benzene ring is lost. This functional group will be unreactive towards base. Choice B is the product of the Hofmann rearrangement shown in Scheme 1, so it is not the product of the competing reaction. Choice C is Compound 1, so shows no reaction at all.

173. B. In this rearrangement, the reactant loses a carbonyl group, so the IR stretch that corresponds to the C=O bond would disappear. Most C=O stretches appear close to 1700 cm–1, so choice B is the best answer.

174. D. In this rearrangement, the reactant loses a carbonyl group, so the 13C NMR signal that corresponds to the carbonyl carbon would disappear. Although ketone carbonyls have signals around 200 ppm, carboxylic acids and their derivatives are nearer 160–170 ppm.

Passage VII (Questions 175–180)

175. B. Typically, a triacylglycerol, or triglyceride, does not contain any stereocenters since two (or more) of the three fatty acids are the same. However, if three different fatty acids are used, the second carbon of the glycerol becomes a stereocenter, because four different substituents are attached.

176. D. Basic hydrolysis of an ester yields an alcohol (the glycerol) and a carboxylate anion (the functional group attached to the fatty acid side chain). Only choice D shows a functional group with the two oxygens of the ester group still present in the product.

177. D. As Reaction 1 shows, a triacylglycerol contains three fatty acid side chains. Eliminate choice A. Since the passage states that the starting compound was a pure sample, it contained a maximum of three fatty acids. Therefore, some fraction of one of the side chains must have been altered during the reaction. Choice C can be eliminated since complete conversion of one fatty acid into another would yield a total of three fatty acids. An alcohol cannot be converted into a carboxylic acid under basic conditions, so choice B can be eliminated.

178. C. An ester group contains a carbonyl attached to an OR group—in this case, the backbone of the glycerol molecule.

179. D. Saponification is not catalyzed by hydroxide since the base becomes incorporated into the carboxylic acid of the fatty acid product. In other words, hydroxide is not regenerated in the reaction, so choice A can be eliminated. Each ester group requires one equivalent of OH– to react, so for each molecule of triglyceride, three OH– ions are required. This means three equivalents of base are necessary to completely saponify the reactant.

180. C. While organic molecules with charges are preferentially soluble in aqueous solution, answer choices A and B are too clear cut. The charged end of a fatty acid salt helps make it soluble in a polar solvent, while the hydrocarbon end helps make it soluble in a nonpolar solvent. The fatty acid salt is therefore amphipathic and can partially dissolve in both environments.

Page 38: AAMC MCAT Test 8 Solnsdocshare01.docshare.tips/files/23478/234782350.pdf · 2016. 12. 13. · MCAT Practice Test 8 Solutions 4 this fact from nuclear physics, you can still choose

MCAT Practice Test 8 Solutions38

Passage VIII (Questions 181–186)

181. C. Testosterone is shown to inhibit the pituitary gland, turning off FSH, which would inhibit the Sertoli cells. In the absence of Sertoli cell activity, sperm production is reduced. The passage states that the effect of testosterone on Sertoli cells is to promote maturation of sperm, not to increase general activity such as release of inhibin, so there is no reason to assume that inhibin concentrations would change (choices A and B are wrong). Testosterone would lead to a reduction in LH secretion (also via inhibition of the pituitary gland), but this would simply lead to a reduction in natural testosterone production, not a reduction in spermatogenesis (choice D is wrong).

182. D. The ovum is the most mature version of the female gamete. It corresponds with the most mature version of the male gamete, the spermatozoon. The spermatogonium is the male version of the oogonium (choice A is wrong), the primary spermatocyte is the male equivalent of the primary oocyte (choice B is wrong), and the spermatid is not a fully mature male gamete (choice C is wrong).

183. C. Spermatogenesis is the only process in the list that requires cell division, so it would be the only process affected by drugs that inhibit cellular proliferation (choice C is correct, and choices A, B, and D are wrong).

184. C. Statement I is false: LH is needed for testosterone production, not directly for sperm production. Statement II is true: FSH is needed to stimulate Sertoli cells, which are needed to promote spermatogenesis. Statement III is false: Inhibin turns off FSH secretion and Sertoli cell activity. Statement IV is true: Testosterone is needed to promote Sertoli cell activity.

185. A. As “nurse” cells, Sertoli cells secrete nutrients, and the Golgi apparatus is necessary for cellular secretion. The lysosomes break down ingested nutrients and old cellular organelles (choice B is wrong), the mitochondria produce ATP for the cell (choice C is wrong), and cilia are used to sweep material along the surface of the cell (choice D is wrong).

186. B. This is the basic difference between exocrine and endocrine tissue: the location of their secreted products. The passage describes the exocrine portion of the testes as sperm production (note that sperm are not peptides, choice A is wrong), which takes place inside coiled ducts called seminiferous tubules. The endocrine portion produces testosterone, a hormone that is released into the blood. The Sertoli cells of the seminiferous tubules (part of the exocrine portion) secrete nutrients, so the exocrine portion does not only secrete cellular elements (choice C is wrong). The exocrine portion is only one of several targets for the product of the endocrine portion; furthermore, it is the target for many other products as well (choice D is wrong).

Free-Standing Questions (187 through 191)

187. D. As far as its digestive role goes, the pancreas produces digestive enzymes (including proteases) and bicarbonate, which it releases into the small intestine. (Note that the pancreas also produces insulin and glucagon, but this is not part of its digestive function.) Bile salts, which are emulsifiers, are produced in the liver (choices A and B are wrong), and gastric juices, by definition, are produced by and released into the stomach (choice C is wrong).

188. C. Mitochondria are only found in eukaryotes. Cell walls and ribosomes are found in both prokaryotes and eukaryotes (choices A and B are wrong), and both prokaryotes and eukaryotes exhibit sexual reproduction (choice D is wrong).

189. A. The products of acidic hydrolysis of an ester are an alcohol and a carboxylic acid. The water acts as a nucleophile, attacking the ester carbonyl, and the oxygen from the water is incorporated into the COOH group. The OR group of the ester, in this case the cyclopentyloxide, becomes the leaving group (choice D). Choices B and C are molecules that are not involved as reactants or products in this addition–elimination reaction.

190. A. Statement I is false: Ectoderm ultimately forms the skin, hair, nails, and nervous system. Statement II is true: Mesoderm forms muscles, bones, blood vessels, and the heart, as well as non-glandular organs, such as the kidneys and the walls of the digestive tract, bladder, and uterus. Statement III is false: Endoderm primarily forms the linings of the digestive system, the respiratory system, and the reproductive system. (Note also though, that it forms the linings of the cardiovascular system, such as the chambers of the heart and the inner linings of the blood vessels, but most of the heart and vessels structure is formed from mesoderm, making choice A a better answer than choice B.)

Page 39: AAMC MCAT Test 8 Solnsdocshare01.docshare.tips/files/23478/234782350.pdf · 2016. 12. 13. · MCAT Practice Test 8 Solutions 4 this fact from nuclear physics, you can still choose

39Biological Sciences Solutions

191. D. Most of the water lost through the skin is lost as sweat, so raising the environmental temperature would lead to an increase in sweating and an increase in water loss. Inhibiting kidney function would prevent water loss through urination, and might promote an increased loss through the skin, but this would be strongly dependent on temperature as well (choice A is wrong). Decreasing salt consumption and increasing water consumption would both have the effect of increasing urinary water loss, but would not affect water loss through the skin (choices B and C are wrong).

Passage IX (Questions 192–197)

192. B. The passage states that the dewlap is part of a visual communication system that involves reflection of UV rays. UV receptors were found in the eyes of lizards involved in this type of communication. The significance of UV would be most easily determined by comparing the behavior of responding lizards that are exposed to reflected UV rays with the behavior of lizards that are not exposed to reflected UV rays (because the dewlap of the flashing lizard has been treated to absorb UV). Comparing sighted and sightless lizards might give information about the significance of visual communication in general, but would not give specific information regarding UV reflection (choice A is wrong). Observing all five lizard species in the same habitat might give information about the response to the amount of UV reflected, but again, would not give information about the significance of reflecting UV rays in the first place (choice C is wrong). Lastly, observing the species under red light illumination would give no information about UV significance at all, since red light is at the opposite end of the spectrum from UV (choice D is wrong).

193. D. Since species E lives in the shaded canopy of the forest, and since it has the lowest reflection of UV radiation by its dewlap, it would probably suffer least from a mutation that prevents detection of UV rays by the eyes. Species A and B both live in the open unshaded fields where plenty of UV radiation is found, so they probably rely more strongly on UV detection (choices A and B are wrong). Lastly, species D, although it also lives in the shaded canopy, reflects more UV rays with its dewlap than does species E, indicating that UV radiation is probably more important to species D than to species E (choice C is wrong).

194. D. Simply knowing that Anolis lizards have X-Y chromosomal sex determination does not give us any further information about the gene for the UV reflectance pigment and where it is found. If UV reflectance were strictly seen in males, the gene for this pigment might be found on the Y chromosome, but this is not a absolute guarantee. In any case, we donʼt know if females also have this pigment and the expression of that gene is simply downregulated, or if itʼs a recessive gene and is carried on the X chromosome (meaning males are more likely to express it). Bottom line: just because a gene is expressed in males, do not automatically assume that the gene must be located on the Y chromosome (choice D is correct, and choices A, B, and C are wrong).

195. D. The definition of “separate species” is that they cannot breed and produce fertile offspring with each other (or with members of any other species, for that matter). The same species can occupy different habitats (owls can live in barns, fields, or forests, choice A is wrong), can have different physical characteristics (some tigers are white and some are orange, choice B is wrong), and may not communicate with one another (for example, it would be difficult for a field mouse in the United States to communicate with a field mouse in Europe even if they were the same species, choice C is wrong). However, if two populations cannot breed or produce fertile offspring, then they are separate species.

196. B. Figure 1 shows that dewlap reflectance is correlated with habitat. All three species that live in unshaded fields have high reflectance, while the two species that live in the shaded canopy have low reflectance (choice B is correct, and choice C is wrong). Note, however, that the correlation does not mean that habitat is determined by dewlap reflectance (choice A is wrong). Figure 1 also shows that dewlap reflectance is actually quite low at the visible blue frequencies, and in some species (E for example) is highest in the green-yellow frequencies (choice D is wrong).

197. A. Evolution is driven by beneficial characteristics that lead to the production of greater numbers of surviving offspring. A characteristic can be beneficial (better communication, less predation, more frequent mating), but if it does not result in greater numbers of surviving offspring, then the characteristic will not be increased by natural selection (choices B, C, and D are wrong).

Page 40: AAMC MCAT Test 8 Solnsdocshare01.docshare.tips/files/23478/234782350.pdf · 2016. 12. 13. · MCAT Practice Test 8 Solutions 4 this fact from nuclear physics, you can still choose

MCAT Practice Test 8 Solutions40

Passage X (Questions 198–204)

198. C. At concentrations of actin greater than 1 μM, the rate of subunit addition to the + end is greater than the rate of addition at the – end. At exactly 1 μM the + end is not growing at all and the – end is losing subunits. Under these conditions the microfilament would be shrinking at the – end (choice A is wrong). At concentrations between 1 μM and 4 μM, the microfilament is definitely growing, but it doesnʼt stop this at concentrations greater than 4 μM (choice B is wrong). At concentrations lower than 1 μM, both the + and the – ends are losing subunits, so the microfilament is shrinking overall (choice D is wrong).

199. B. If both ends of the microfilament are capped, no growth is possible. If the – ends are capped by the Z lines and the actin concentration is greater than 1 μM, then the uncapped + ends would continue to grow and the microfilaments would get longer (choice A is wrong). If the actin concentration were kept above 4 μM, then both the + and – ends would be growing (choice C is wrong). If the – ends grow at the same rate that the + ends shrink, then the microfilament is treadmilling but is not stable (choice D is wrong).

200. A. The theory of force generation states that microfilaments can generate force by growing and pushing against the plasma membrane of a cell. Cytochalasin prevents growth of the microfilaments and, if the theory is correct, should stop the pushing of the plasma membrane that is seen in pseudopod formation (that is, amoeboid movement). Mitosis involves the use of microtubules, not microfilaments, so blocking mitosis and movement would not support the theory of force generation by microfilaments (choice B is wrong). Troponin does not play a role in the theory of force generation (choice C is wrong), nor is anything stated about the viscosity of the medium (choice D is wrong). 201. C. Figure 1 shows that when the actin concentration is 1.5 μM, the rate of addition of subunits to the + end is exactly equal to the rate of removal of subunits from the – end, and this is defined in the passage as “treadmilling.” At 0.25 μM both ends are losing subunits (choice A is wrong), at 1.0 μM the + end is stable but the – end is losing subunits (choice B is wrong), and at concentrations between 1 μM and 4 μM, the + end is gaining subunits while the – end is losing them; however, the rate of addition is much greater than the rate of loss, and the microfilament would be growing toward the + end (choice D is wrong).

202. A. Any actin concentration below 1 μM results in a loss of subunits from both ends. At exactly 1.0 μM the – end is losing subunits, but the + end is stable. Note that this is a net loss, but that the loss of subunits at concentrations below 1 μM is much greater because both ends are losing subunits (choice A is better than choice B). At concentrations above 1 μM, the rates of loss and gain vary, and in some cases both ends are gaining subunits (choice C is wrong). At concentrations between 1 μM and 4 μM, the + end is gaining subunits while the – end is losing them; however, the rate of addition is much greater than the rate of loss, and the microfilament would be growing toward the + end (choice D is wrong).

203. A. If exocytosis of viruses and microfilament polymerization are proportional, then they could be linked. If treatment with phalloidin, which prevents microfilament growth, does not stop the release of viruses, then the two processes are independent (choice B is wrong). Viruses would not need to carry genes for actin subunits, since those would be coded for by the host cell (choice C is wrong). Capsules made of myosin might support the use of microfilaments in viral release, but linking the rates of exocytosis and microfilament polymerization (as in choice A) is better evidence (choice D is wrong).

204. D. At actin concentrations of 2 μM, the + end on the right is adding subunits faster than the – end on the left is losing them, causing the microfilament to grow towards the right. This would push the Amoeba towards the right. At 0.5 μM both the + end and the – end would lose subunits, and the Amoeba would be going nowhere (choice A is wrong). At 6 μM both the + end and the – end would be gaining subunits, and the Amoeba would be stretching in both directions (choice B is wrong). At 1 μM the + end would be stable and the – end would be losing subunits, and the Amoeba would be shrinking at the left (choice C is wrong).

Passage XI (Questions 205–208)

205. A. Equation 1 in the passage shows the migration of a methyl group, an example of a rearrangement. In order for a skeletal rearrangement to occur, a carbocation intermediate must be produced. Only the addition of a proton from the H2SO4 to the alkene yields such an intermediate.

Page 41: AAMC MCAT Test 8 Solnsdocshare01.docshare.tips/files/23478/234782350.pdf · 2016. 12. 13. · MCAT Practice Test 8 Solutions 4 this fact from nuclear physics, you can still choose

41Biological Sciences Solutions

206. C. Equation 3 in the passage shows the addition of the OH group to the least substituted carbon of the double bond. This corresponds to the outcome of the reaction in the question. Both choices A and B would add the OH group to the most substituted carbon of the starting material, while the passage indicates nothing about heat accomplishing a hydration reaction with no other reagents present.

207. A. Molecules containing OH groups, as alcohols do, can hydrogen bond. Hydrogen bonds are the strongest of the intermolecular interactions (stronger than van der Waals forces in choice B). This means they have strong attractions, requiring more energy to vaporize them. Choices C and D relate to intramolecular interactions, which do not determine physical properties of molecules such as boiling points.

208. A. Grignard reagents are added to carbonyl compounds to produce alcohols as products. Therefore, choices B and C can be eliminated. In the presence of alcohols, Grignard reagents will act as bases and deprotonate the OH group. In order to form a tertiary alcohol product, a ketone starting material is required. The aldehyde in choice D will yield a secondary alcohol product.

Free-Standing Questions (209 through 214)

209. A. Watson and Crick proposed the complementary theory of base pairing, A with T and G with C. The finding that the amounts of these bases were proportional supported their theory. This does not give any evidence for DNA being genetic material (choice B is wrong), for the genetic code being the same in all organisms (choice C is wrong), or for a triplet genetic code (choice D is wrong).

210. B. The number of different gametes that can be produced is equal to 2n, where n is the number of heterozygous alleles the organism possesses. In this case, the organismʼs genotype is AaBbCc. Since it possesses 3 heterozygous pairs of alleles, the number of different gametes it can produce is 23 = 8.

211. C. If the tidal volume (amount of air moved in a single breath) is 800 mL and the dead space volume is 150 mL, then the amount of air reaching the alveoli is only 650 mL. If the breathing rate is 10 breaths/min, then the amount of air reaching the alveoli is 650 mL/breath × 10 breaths/min = 6500 mL/min.

212. B. Splitting of centromeres (separation of sister chromatids) happens in mitosis and in meiosis II, but not in meiosis I. Synapsis (pairing of homologs) occurs only in meiosis I (choices A and C are wrong). Breaking down of the nuclear membrane occurs in both mitosis and meiosis I (choice D is wrong).

213. D. The tall pink plant heterozygous for height has the genotype TtRr. The short pink plant has the genotype ttRr. The probability of getting a pink plant (a heterozygote) by crossing two heterozygotes is 1/2, and the probability of getting a tall plant by crossing Tt and tt is also 1/2. Thus, using the Rule of Multiplication, to get a plant that is pink and tall is 1/2 × 1/2 = 1/4.

214. D. Eukaryotes transcribe RNA in the nucleus and translate proteins in the cytosol, so the processes cannot occur simultaneously. Both eukaryotes and prokaryotes use ribosomes for protein translation (choice A is wrong), and both use DNA to transcribe mRNA (choice B is wrong). The rate of destruction of mRNA varies depending on the mRNA (choice C is wrong).